All MBE Topics Flashcards

1
Q

Reply Letter Doctrine

A

A document may be authenticated by evidence that it was written in response to a communication, so long as it is unlikely, based on the contents, that it was written by someone other than the recipient of the first communication.

How well did you know this?
1
Not at all
2
3
4
5
Perfectly
2
Q

What must a government employee show when she contends that her rights under the Free Speech Clause of the First Amendment have been violated by her employer?

A

When a government employee contends that her rights under the Free Speech Clause of the First Amendment have been violated by her employer, the employee must show that she was speaking as a citizen on a matter of public concern.

When a government employee is speaking pursuant to her official duties, the employee is generally not speaking as a citizen and the Free Speech Clause does not protect the employee from employer discipline.

In determining whether a government employee is speaking pursuant to her official duties, the critical question is whether the speech at issue is itself ordinarily within the scope of an employee’s duties, not whether it merely concerns those duties.

How well did you know this?
1
Not at all
2
3
4
5
Perfectly
3
Q

How many days after filing a complaint must the summons and complaint be served?

A

Unless service is made in a foreign country, the summons and complaint generally must be served within 90 days after filing the complaint.

How well did you know this?
1
Not at all
2
3
4
5
Perfectly
4
Q

Does an offender have an absolute constitutional right to counsel at a probation revocation hearing when an already-imposed sentence is executed as a result of the revocation of probation?

A

NO.

An offender does not have an absolute constitutional right to counsel at a probation revocation hearing when an already-imposed sentence is executed as a result of the revocation of probation.

  • Instead, an offender only has such a right if it is necessary for a fair trial.
How well did you know this?
1
Not at all
2
3
4
5
Perfectly
5
Q

Collateral-order Doctrine

A

Under the collateral-order doctrine, a court of appeals has discretion to hear and rule on a district court order if it:

(i) conclusively determines the disputed question,
(ii) resolves an important issue that is completely separate from the merits of the action, and
(iii) is effectively unreviewable on appeal from a final judgment.

How well did you know this?
1
Not at all
2
3
4
5
Perfectly
6
Q

Can a grant of immunity from prosecution, such as the immunity provided by the Eleventh Amendment, fall within the narrow confines of the collateral-order doctrine?

A

YES

A grant of immunity from prosecution, such as the immunity provided by the Eleventh Amendment, can fall within the narrow confines of the collateral-order doctrine because the benefit of immunity would be lost if the party claiming it were wrongfully forced to proceed to trial.

How well did you know this?
1
Not at all
2
3
4
5
Perfectly
7
Q

Are answers to interrogatories nonhearsay admissions?

A

YES

Answers to interrogatories, which are signed and made under oath by a party to the litigation, are nonhearsay admissions.

example: Because an interrogatory by the manufacturer included by reference the content of the identified patent, the patent will also be admissible as an adoptive admission by the manufacturer.

How well did you know this?
1
Not at all
2
3
4
5
Perfectly
8
Q

Can conspiracy be based solely on knowledge of the existence of the conspiracy?

A

NO

Criminal conspiracy requires not only an intent to agree but also an intent to commit the crime, criminal liability for conspiracy cannot be based solely on knowledge of the existence of the conspiracy.

How well did you know this?
1
Not at all
2
3
4
5
Perfectly
9
Q

What type of SMJ is required when the cause of action is based on state common law rather than federal question?

A

Diversity Jurisdiction

When the cause of action is based on state common law rather than a federal question, subject matter jurisdiction exists on the basis of diversity of citizenship when the plaintiff and the defendant are U.S. citizens as well as citizens of different states.

Example:

Here, although the individual and the corporation are both U.S. citizens, and the corporation is a citizen of both State A where its principal headquarters is located and State B where it is incorporated, the individual is not a citizen of any U.S. state, but instead, having moved permanently to a foreign country, is domiciled in the foreign country.

Therefore, there is no diversity of citizenship because the action is not between citizens of different States. Moreover, alienage jurisdiction is lacking because the individual is not an alien: the facts indicate that the individual has retained his U.S. citizenship. Consequently, subject matter jurisdiction does not exist.

How well did you know this?
1
Not at all
2
3
4
5
Perfectly
10
Q

When a hearsay statement is admitted into evidence, can the credibility of the declarant be attacked (and, if attacked, supported) by any evidence that would be admissible if the declarant had testified as a witness?

A

YES

How well did you know this?
1
Not at all
2
3
4
5
Perfectly
11
Q

What are the 5 exceptions to the hearsay rule that apply only if the declarant is unavailable as a witness?

A

There are five exceptions to the hearsay rule that apply only if the declarant is unavailable as a witness:

1) former testimony,
2) dying declaration,
3) statement against interest,
4) statement of personal or family history, AND
5) statement offered against a party that wrongfully caused the declarant’s unavailability.

How well did you know this?
1
Not at all
2
3
4
5
Perfectly
12
Q

When a jury renders a verdict that a defendant is guilty of certain offenses but not guilty of other related offenses, is the guilty verdict reviewable on the grounds of inconsistency, even when the jury acquits the defendant of an offense that is a predicated on an offense for which the same jury finds the defendant guilty?

A

NO

When a jury renders a verdict that a defendant is guilty of certain offenses but not guilty of other related offenses, the guilty verdict is not reviewable on the grounds of inconsistency, even when the jury acquits the defendant of an offense that is a predicated on an offense for which the same jury finds the defendant guilty.

How well did you know this?
1
Not at all
2
3
4
5
Perfectly
13
Q

When is silence in response to a statement considered to be an adoptive admission?

A

Silence in response to a statement is considered an adoptive admission if:

(i) the person was present and heard and understood the statement,
(ii) the person had the ability and opportunity to deny the statement, and
(iii) a reasonable person similarly situated would have denied the statement.

How well did you know this?
1
Not at all
2
3
4
5
Perfectly
14
Q

Is a statement of an opposing party admissible regardless of whether the opposing party is available as a witness?

A

YES.

How well did you know this?
1
Not at all
2
3
4
5
Perfectly
15
Q

When does the First Amendment shield the media from liability for publishing illegally obtained information?

A

The press has the right to publish information about matters of public concern, and the viewers have a right to receive it.

The First Amendment shields the media from liability for publishing information that was obtained illegally by a third party as long as the information:

i) involves a matter of public concern and

ii) the publisher did not obtain it unlawfully.

How well did you know this?
1
Not at all
2
3
4
5
Perfectly
16
Q

When may a party voluntariily withdraw a cross-claim (or counterclaim or third-party claim) without the approval of the court or the consent of the parties?

A

A party may voluntarily withdraw a cross-claim (or a counterclaim or third-party claim) without the approval of the court or the consent of the parties before a responsive pleading is served, or if there is no responsive pleading, before evidence is introduced at a hearing or trial.

NOTE: The service of a summary judgment motion by a co-party with respect to a cross-claim (or counterclaim or third-party claim) does not cut off a party’s ability to voluntarily dismiss the claim without court approval or the consent of the other parties.

  • This occurs only after evidence is introduced at the court hearing on the motion.
How well did you know this?
1
Not at all
2
3
4
5
Perfectly
17
Q

How can a motion to dismiss for failure to state a claim upon which relief can be granted be converted into a summary judgment motion?

A

Through the attachment of an affidavit presenting evidence beyond the pleadings.

How well did you know this?
1
Not at all
2
3
4
5
Perfectly
18
Q

Can nonresidents of a city be prohibited from voting if they do not reside within the borders?

A

YES

A restriction on the right to participate in the political process of a governmental unit to those who reside within its borders is typically upheld as justified on a rational basis.

Nonresidents generally may be prohibited from voting.

The restriction on the right to participate in the political process of a governmental unit, including a city, may preclude those who do not reside within its borders, even though they may be directly affected by decisions made by city officials, such as a licensing fees imposed on those who operate a business within the city.

How well did you know this?
1
Not at all
2
3
4
5
Perfectly
19
Q

False pretenses requires:

A

False pretenses requires:

(i) obtaining title to the property
(ii) of another person
(iii) through the reliance of that person
(iv) on a known false representation of a material past or present fact, and
(v) the representation is made with the intent to defraud.

How well did you know this?
1
Not at all
2
3
4
5
Perfectly
20
Q

Under diversity jx, does federal law or state law govern whether there is a right to a jury trial?

A

Because the Seventh Amendment provides for a right to a jury trial, federal law governs whether there is a right to a jury trial, even when the action is based on state law and the federal court’s jurisdiction is based on diversity.

A party may specify the issues for which it is demanding a jury trial.

Example: In a mbe question jx, under federal law, punitive damages are determined by the jury. The plaintiff can demand a jury trial with regard to the issue of punitive damages. Consequently, the court should deny the defendant’s motion to strike the jury trial demand.

How well did you know this?
1
Not at all
2
3
4
5
Perfectly
21
Q

Who can identify a voice?

A

A voice can be identified by any person who has heard the voice at any time. The fact that a person was not aware of the speaker’s identity at the time she heard the statements does not prevent her from identifying the man as their speaker at trial.

How well did you know this?
1
Not at all
2
3
4
5
Perfectly
22
Q

Can a party serve interrogatories before the parties have held a Rule 26(f) discovery conference?

A

NO. Such interrogatories are premature.

How well did you know this?
1
Not at all
2
3
4
5
Perfectly
23
Q

What must a defendant establish in order to successfully challenge a facially valid warrant?

A

A defendant can successfully challenge a facially valid warrant only when the defendant can establish, by a preponderance of the evidence, that:

(i) the affidavit contained false statements that were made by the affiant knowingly, intentionally, or with a reckless disregard for their truth; and
(ii) the false statements were necessary to the finding of probable cause.

How well did you know this?
1
Not at all
2
3
4
5
Perfectly
24
Q

Is testimony given under a grant of immunity considered coerced and involuntary?

Can such a testimony be used substantively or for impeachment purposes?

A

YES.

Testimony given under a grant of immunity is considered coerced and involuntary.

A defendant’s involuntary statement, such as a confession produced by coercion, cannot be used either substantively or for impeachment purposes.

How well did you know this?
1
Not at all
2
3
4
5
Perfectly
25
Q

Rule 14: Third-party claims (impleader)

A

Third-party claims (impleader) are claims made by a defending party against a nonparty for all or part of the defending party’s liability on an original claim.

The impleaded claim must relate to the original claim against the defending party.

In judging whether the claims are related, the test is whether they arise out of a “common nucleus of operative fact” such that all claims should be tried together in a single judicial proceeding.

How well did you know this?
1
Not at all
2
3
4
5
Perfectly
26
Q

Joinder of Parties

  • Rule 20 - Permissive*
  • Rule 19 - Compulsory*
A

The Federal Rules of Civil Procedure provide for joining parties to existing litigation, generally for reasons of efficiency and economy. Joinder may be permissive (pursuant to Rule 20) or compulsory (pursuant to Rule 19).

Rule 19 specifies circumstances in which additional parties must be joined.

Note that the requirements of jurisdiction (both subject matter and personal) and venue must still be met in order for compulsory joinder to occur.

Under certain circumstances, if compulsory joinder cannot occur because of jurisdictional or venue issues, Rule 19(b) may require the action to be dismissed from federal court.

How well did you know this?
1
Not at all
2
3
4
5
Perfectly
27
Q

Rule 20 - Permissive Joinder

A

Rule 20 sets forth the circumstances in which a plaintiff may join other plaintiffs in an action or in which defendants may be joined in the same action.

  1. Plaintiffs

Pursuant to Rule 20(a)(1), persons may join in one action as plaintiffs if:

i) They assert any right to relief jointly, severally, or in the alternative with respect to or arising out of the same transaction, occurrence, or series of transactions or occurrences; and
ii) Any question of law or fact common to all plaintiffs will arise in the action.
2. Defendants

Pursuant to Rule 20(a)(2), persons may be joined in one action as defendants if:

i) Any right to relief is asserted against them jointly, severally, or in the alternative with respect to or arising out of the same transaction, occurrence, or series of transactions or occurrences; and
ii) A question of law or fact common to all defendants will arise in the action.

A plaintiff or defendant sought to be joined must also meet the requirements of federal subject matter jurisdiction.

How well did you know this?
1
Not at all
2
3
4
5
Perfectly
28
Q

Rule 19 - Compulsory Joinder

A
  1. Necessary Parties

Under Rule 19(a), a person who is subject to service of process and whose joinder will not deprive the court of subject matter jurisdiction or destroy venue must be joined as a party if:

i) Complete relief cannot be provided to existing parties in the absence of that person; or
ii) Disposition in the absence of that person may impair the person’s ability to protect his interest; or
iii) The absence of that person would leave existing parties subject to a substantial risk of multiple or inconsistent obligations.

How well did you know this?
1
Not at all
2
3
4
5
Perfectly
29
Q

Rule 19 - Compulsory Joinder

Who is considered a necessary party?

A

A necessary party is therefore a person whose participation in the lawsuit is necessary for a just adjudication.

Tortfeasors facing joint and several liability are not parties who must be joined under Rule 19.

A plaintiff or defendant to be joined under Rule 19 must meet the requirements of federal subject matter jurisdiction.

Thus, if the exclusive basis for the court’s subject matter jurisdiction is diversity jurisdiction, and a party sought to be joined would destroy diversity, joinder is not permitted.

How well did you know this?
1
Not at all
2
3
4
5
Perfectly
30
Q

Does supplemental jx apply to the claims of a party sought to be joined under Rule 19 in a diversity action?

venue?

A

Only if the exercise of jurisdiction would be consistent with the diversity requirements.

Supplemental jurisdiction does not apply to the claims of a party sought to be joined under Rule 19 in a case based exclusively on diversity jurisdiction if the exercise of jurisdiction would be inconsistent with the diversity requirements.

If a joined party objects to venue and the joinder would make venue improper, the court must dismiss that party.

How well did you know this?
1
Not at all
2
3
4
5
Perfectly
31
Q

What must the court determine when joinder of indispensable parties is not feasible?

A

Under Rule 19(b), if a necessary party cannot be joined because of jurisdictional or venue concerns, then the court must determine whether, in equity and good conscience, the action should proceed among the existing parties or be dismissed.

Among the factors for the court to consider are:

i) The extent to which a judgment rendered in the person’s absence might prejudice that person or the existing parties;
ii) The extent to which any prejudice could be reduced or avoided by protective provisions in the judgment, shaping the relief, or other measures;
iii) Whether a judgment rendered in the person’s absence would be adequate; and
iv) Whether the plaintiff would have an adequate remedy if the action were dismissed for nonjoinder.

When the court dismisses an action because of the inability to join a necessary party, the party is said to be “indispensable.”

How well did you know this?
1
Not at all
2
3
4
5
Perfectly
32
Q

When is strict liability for an abnormally dangerous activity available?

A

Strict liability for an abnormally dangerous activity is available if the harm that actually occurs results from the risk that made the activity abnormally dangerous in the first place.

How well did you know this?
1
Not at all
2
3
4
5
Perfectly
33
Q

Common law Arson

A

Common-law arson is the malicious burning of the dwelling of another.

The crime of arson is a malice crime requiring a reckless disregard of a high risk of harm.

How well did you know this?
1
Not at all
2
3
4
5
Perfectly
34
Q

Can the government seize private property in order to transfer it to another private party?

A

YES. Such a seizure is permissible if it is rationally related to a conceivable public purpose.

The Fifth Amendment Takings Clause, which applies to the states through the Fourteenth Amendment, provides that a government may seize private property not only for its own direct use, but also in order to transfer the property to another private party.

How well did you know this?
1
Not at all
2
3
4
5
Perfectly
35
Q

When does jeopardy attach?

A

Jeopardy does not attach until a trial begins. Therefore, the Double Jeopardy Clause does not apply to grand jury proceedings.

How well did you know this?
1
Not at all
2
3
4
5
Perfectly
36
Q

Does the privilege against self-incrimination extend to both criminal and civil proceedings?

A

The privilege against self-incrimination extends to a witness in any proceeding, whether civil or criminal, formal or informal, if the answers provide some reasonable possibility of incriminating the witness in future criminal proceedings.

How well did you know this?
1
Not at all
2
3
4
5
Perfectly
37
Q

Does a fraudulent misrepresentation give the person defrauded the chance to avoid a K?

A

YES

A fraudulent misrepresentation gives the person defrauded the chance to avoid a contract with the person who made the fraudulent assertion.

Nondisclosure of a known fact is tantamount to an assertion that the fact does not exist, if the party not disclosing the fact knows that disclosure would correct a mistake of the other party as to a basic assumption of the contract, and the failure to disclose would constitute lack of good faith and fair dealing.

How well did you know this?
1
Not at all
2
3
4
5
Perfectly
38
Q

What does the competency of a child to testify depend on?

A

The competence of a child depends on his intelligence, his ability to differentiate between truth and falsehood, and his understanding of the importance of telling the truth.

A witness who is so young that he is unable to understand the requirement to tell the truth would be disqualified.

How well did you know this?
1
Not at all
2
3
4
5
Perfectly
39
Q

A plaintiff may bring an action for defamatioin if:

A

i) The defendant’s defamatory language;
ii) Is of or concerning the plaintiff;
iii) Is published to a third party who understands its defamatory nature; and
iv) It damages the plaintiff’s reputation.

For matters of public concern, the plaintiff is constitutionally required to prove fault on the part of the defendant.

If the plaintiff is either a public official or a public figure, then the plaintiff must prove actual malice.

If either (i) the defamatory statement relates to a matter of public concern or (ii) the plaintiff is a public official or a public figure, then the plaintiff must prove that the defamatory statement is false as part of her prima facie case.

How well did you know this?
1
Not at all
2
3
4
5
Perfectly
40
Q

What is a prior restraint?

A

A prior restraint is a regulation of speech that occurs in advance of its expression (e.g., publication or utterance).

In prior restraint cases, the burden is on the government to prove that the material to be censored is not protected speech.

How well did you know this?
1
Not at all
2
3
4
5
Perfectly
41
Q

What constitutes publication for purposes of defamation?

A

Publication is the intentional or negligent communication to a third party (i.e., to someone other than the person being defamed) who understands its defamatory nature.

Example: “In this case, the supervisor did not successfully communicate anything to a third party because the potential employer did not hear the statements. Rather, the only person that heard the statement by the supervisor was the woman herself.”

How well did you know this?
1
Not at all
2
3
4
5
Perfectly
42
Q

Crime of Possession

A

In order to be found guilty of possession, the defendant must exhibit dominion and control over the prohibited object. Generally, an illegal substance found on a defendant’s person is sufficient evidence that the defendant has exercised control over the illegal substance.

Dominion and control must exist for a period long enough to have provided the defendant with an opportunity to cease such dominion and control.

Example:

Here, the prosecution failed to establish the necessary actus reus (i.e., possession) for this crime.

If the man’s testimony is believed, the drugs were not in the suitcase when he surrendered it to the airline.

The man had not regained possession of the suitcase at the time it was seized and searched by the police officer. Therefore, he never exhibited dominion and control over the prohibited substance.

How well did you know this?
1
Not at all
2
3
4
5
Perfectly
43
Q

What award can a court give for a breach of construction contract if the award of damages based on the cost to fix or complete the construction would result in economic waste?

A

When breach of a construction contract results in a defective or unfinished construction, if the award of damages based on the cost to fix or complete the construction would result in economic waste, then a court has the discretion to award damages equal to the diminution in the market price of the property caused by the breach.

Economic waste occurs when the cost to fix or complete the construction is clearly disproportional to any economic benefit or utility gained as a result.

However, if the breach is willful, and only completion of the contract will give the nonbreaching party the benefit of its bargain, then a court may award damages based on the cost to fix or complete the construction, even if that award would result in economic waste.

How well did you know this?
1
Not at all
2
3
4
5
Perfectly
44
Q

When can a private citizen make an arrest?

A

A private citizen is privileged to use force to make an arrest in the case of a felony if the felony has in fact been committed and the arresting party has reasonable grounds to suspect that the person being arrested committed it.

It is a defense to make a reasonable mistake as to the identity of the felon.

How well did you know this?
1
Not at all
2
3
4
5
Perfectly
45
Q

When does a taking occur?

A

A taking has occurred when a government’s action results in a permanent physical occupation of private property by the government or a third party.

How well did you know this?
1
Not at all
2
3
4
5
Perfectly
46
Q

Under the majority rule, when is withdrawal from conspiracy possible?

A

Under the majority rule, a conspiracy does not exist until an overt act has occurred, and withdrawal is possible between the date of the agreement and the commission of the overt act.

  • In order to withdraw, notice must be communicated to the other co-conspirators, or the police must be advised of the existence of a conspiracy in a timely manner.

Upon completion of the overt act (here, renting the van), the conspiracy is formed, and withdrawal is no longer possible.

How well did you know this?
1
Not at all
2
3
4
5
Perfectly
47
Q

Under the majority and MPC rule, when is withdrawal as an accomplice possible?

A

Under the majority and MPC rule, an accomplice is a person who, with the purpose of promoting or facilitating the commission of the offense, aids or abets a principal prior to or during the commission of the crime.

  • An accomplice is responsible for the crime to the same extent as the principal.

To legally withdraw (and therefore avoid liability for the substantive crime), the accomplice must repudiate prior aid or do all that he can to negate the prior assistance before the crime is put into motion.

  • A mere change of heart after the crime is put into motion is ineffective, as notification to the legal authorities must be timely and directed toward preventing others from committing the crime.
How well did you know this?
1
Not at all
2
3
4
5
Perfectly
48
Q

Does the physician-patient privilege prevent a physician from testifying about an observation made by the physician of the patient’s condition?

A

Probably not.

Most states recognize a physician-patient privilege that protects communications made by a patient to a physician for the purpose of obtaining treatment.

However, if the testimony of the emergency room physician does not concern a communication made by the patient to the physician, but instead involves an observation by the physician of the patient’s condition, then, consequently, it is likely that the defendant-patient cannot prevent the physician from testifying as to as to his inebriated condition when he was brought to the emergency room.

How well did you know this?
1
Not at all
2
3
4
5
Perfectly
49
Q

Does the President enjoy immunity for civil liability from conduct that occurred before he/she took office?

A

NO.

A President does not enjoy immunity from an action for civil liability that stems from conduct that occurred before the president took office.

In this regard, the President is subject to litigation in the same manner as any private citizen.

How well did you know this?
1
Not at all
2
3
4
5
Perfectly
50
Q

Right to Speedy Trial Factors

A

The factors to be considered in determining whether the defendant has been deprived his post-accusation right to a speedy trial are:

(i) the length of the delay;
(ii) the reason for the delay;
(iii) the defendant’s assertion of a right to a speedy trial; and
(iv) the prejudice to the defendant resulting from the delay.

How well did you know this?
1
Not at all
2
3
4
5
Perfectly
51
Q

Right to Speedy Trial

A

The Due Process Clause and federal and state statutes protect defendants from intentional and prejudicial pre-accusation delay.

The Sixth Amendment speedy trial guarantee, the Fourteenth Amendment, the Speedy Trial Act of 1974, and other federal and state statutes protect defendants from undue post-accusation delay.

How well did you know this?
1
Not at all
2
3
4
5
Perfectly
52
Q

Commencement of the Right to Speedy Trial

A

Statutes of limitations are the primary safeguards against pre-accusation delay. However, the Due Process Clause may be violated if the delay was used to obtain a tactical advantage for the prosecution or to harass the defendant. Delay resulting from an investigation conducted in good faith does not violate the Due Process Clause.

Under the Sixth and Fourteenth Amendments, the time period commences at the time of arrest or formal charge, whichever comes first. The defendant need not know about the charges against him for the right to attach.

How well did you know this?
1
Not at all
2
3
4
5
Perfectly
53
Q

Private Nuisance

A

A private nuisance is a substantial, unreasonable interference with another individual’s use or enjoyment of his property.

The interference may be:

i) intentional,
ii) negligent,
iii) reckless, or
iv) the result of abnormally dangerous conduct.

How well did you know this?
1
Not at all
2
3
4
5
Perfectly
54
Q

Can parties to a contract eliminate liability for consequential damages?

A

YES.

Although a party may be liable for consequential damages of the other party to a contract where those damages are foreseeable, a party may eliminate that liability through an agreement with the other party.

although the UCC does prohibit the limitation or exclusion of liability for consequential damages, this prohibition applies only where limitation or exclusion is unconscionable, and a commercial loss is not prima facie unconscionable.

How well did you know this?
1
Not at all
2
3
4
5
Perfectly
55
Q

Is the owner of a wild animal strictly liable to a trespasser who is injured by the wild animal?

A

NO. Except for injuries caused by a vicious watchdog.

How well did you know this?
1
Not at all
2
3
4
5
Perfectly
56
Q

Impeachment by evidence of crime involving dishonesty or false statement?

A

Any witness may be impeached with evidence that he has been convicted of any crime—felony or misdemeanor—involving dishonesty or false statement, regardless of the punishment imposed or the prejudicial effect of the evidence, provided that no more than 10 years have lapsed since the later of the date of conviction or the release from confinement.

How well did you know this?
1
Not at all
2
3
4
5
Perfectly
57
Q

Does the 403 Balancing test apply to the impeachment of a witness with a conviction of a crime involving dishonesty or false statement?

A

NO.

The Rule 403 test does not apply to the impeachment of a witness with a conviction of a crime involving dishonesty or false statement.

Also note that the special test for impeachment of criminal defendant who testifies (i.e., the probative value of the conviction outweighs its prejudicial effect) does not apply to a conviction of a crime involving dishonesty or false statement.

How well did you know this?
1
Not at all
2
3
4
5
Perfectly
58
Q

If the police have no reasonable suspicion that a car contains drugs, can they conduct a dog sniff that prolongs an otherwise valid traffic stop beyond the time reasonably needed to complete the stop?

A

NO.

If the police have no reasonable suspicion that a car contains drugs, a dog sniff that prolongs an otherwise valid traffic stop beyond the time reasonably needed to complete the stop constitutes an unreasonable search.

How well did you know this?
1
Not at all
2
3
4
5
Perfectly
59
Q

When is a defendant liable for conversion?

A

A defendant is liable for conversion if he intentionally commits an act depriving the plaintiff of possession of his chattel or interfering with the plaintiff’s chattel in a manner so serious as to deprive the plaintiff of the use of the chattel.

The plaintiff’s damages are the chattel’s full value at the time of the conversion.

  • The defendant must only intend to commit the act that interferes; intent to cause damage is not necessary.
  • Accidentally damaging the plaintiff’s chattel is not conversion if the defendant had permission to be using the property.
How well did you know this?
1
Not at all
2
3
4
5
Perfectly
60
Q

Replevin

A

Replevin is a remedy to conversion.

The plaintiff may recover damages in the amount of the full value of the converted property at the time of the conversion. Alternatively, the plaintiff may bring an action for replevin to recover the chattel.

How well did you know this?
1
Not at all
2
3
4
5
Perfectly
61
Q

Does a possibility of reverter in a property render a title unmarketable?

A

YES.

How well did you know this?
1
Not at all
2
3
4
5
Perfectly
62
Q

To establish a prima facie case for non-representative jury selection, a defendant must show that:

A

(i) the group excluded is a distinctive group in the community;
(ii) the group was not fairly represented in the venire from which the jury was selected; and
(iii) the underrepresentation resulted from a systematic exclusion of the group.

How well did you know this?
1
Not at all
2
3
4
5
Perfectly
63
Q

Is the right to have a jury selected from a representative cross-section of the community subject to harmless error analysis?

A

NO.

The right to have a jury selected from a representative cross-section of the community is not subject to harmless error analysis.

How well did you know this?
1
Not at all
2
3
4
5
Perfectly
64
Q

Ademption

A

A devise of real property may fail (or be “adeemed”) because the testator no longer owns the property upon death (i.e., because the property was sold, destroyed, or given away before death).

If the testator gives the property to the intended beneficiary while the testator is still alive, then the devise is adeemed by satisfaction. Once a devise is adeemed, the beneficiary named in the will takes nothing.

EXAM NOTE: Remember that ademption can occur only with a specific devise, such as real property, and not with a general devise, such as money.

Example: A executes a will under which Blackacre is devised to B, but then he sells Blackacre to C. When A dies, the gift is adeemed, and B takes nothing.

How well did you know this?
1
Not at all
2
3
4
5
Perfectly
65
Q

Conveyance in Will

When does lapse occur?

A

Lapse occurs when the intended beneficiary predeceases the testator.

How well did you know this?
1
Not at all
2
3
4
5
Perfectly
66
Q

When can states tax interstate commerce?

A

States may tax interstate commerce if Congress has not already acted in the particular area and if the tax does not discriminate against or unduly burden interstate commerce.

The Supreme Court applies a four-part test to determine whether a state tax comports with the Commerce Clause:

(1) there must be a substantial nexus between the activity taxed and the taxing state,
(2) the tax must be fairly apportioned according to a rational formula,
(3) the tax may not provide a direct commercial advantage to local businesses over interstate competitors, and
(4) there must be a fair relationship between the tax and the service provided.

How well did you know this?
1
Not at all
2
3
4
5
Perfectly
67
Q

When is a checkpoint for the purpose of finding witnesses to a crime not per se unreasonable?

A

A checkpoint maintained by police for the purpose of finding witnesses to a crime (rather than suspects) is not per se unreasonable, as long as:

(i) the checkpoint stop’s primary law enforcement purpose is to elicit evidence to help them apprehend not the vehicle’s occupants but other individuals;
(ii) the stop advanced a public concern to a significant degree; and
(iii) the police appropriately tailored their checkpoint stops to fit important criminal investigatory needs and to minimally interfere with liberties protected by the Fourth Amendment.

Officers must generally have a reasonable, individualized suspicion of a violation of the law to justify the stop of a vehicle.

However, police officers may stop an automobile at a checkpoint without such suspicion if the stop is based on neutral, articulable standards and its purpose is closely related to an issue affecting automobiles.

How well did you know this?
1
Not at all
2
3
4
5
Perfectly
68
Q

Will an express condition in a contract be excused by substantial performance by the other party?

A

NO. Express conditions must be complied with fully unless excused; substantial performance will not suffice.

Express conditions are expressed in the contract. Words in the contract such as “on the condition that” or “provided that” are typical examples of express conditions.

How well did you know this?
1
Not at all
2
3
4
5
Perfectly
69
Q

Mortgages

Subrogation (Payment by a Third Party)

A

A person who pays off another person’s mortgage obligation may become the owner of the obligation and the mortgage to the extent necessary to prevent unjust enrichment.

Among the circumstances in which the equitable remedy of subrogation is appropriate is when the payor (i.e., the subrogee) is under a legal duty to pay the obligation, or when the payor does so to protect his own interest or on account of misrepresentation, mistake, duress, fraud, or undue influence.

  • Subrogation is not permitted when the full obligation secured by the mortgage is not discharged.

(Note: An obligation may be fully discharged even though the payor pays less than the face value of the obligation if the payor does so as a result of a negotiated settlement with the obligee.)

In the case of a partial discharge, a payor who is a subordinate mortgagee may be able to add the amount paid to the balance of the subordinate mortgage and recover the amount upon foreclosure.

How well did you know this?
1
Not at all
2
3
4
5
Perfectly
70
Q
A
How well did you know this?
1
Not at all
2
3
4
5
Perfectly
71
Q

Venue is proper in any federal district where:

A

(1) any defendant resides, so long as all defendants reside in the same state,
(2) a substantial part of the events occurred or the property at issue is located, or
(3) any defendant is subject to personal jurisdiction—but only if the first two provisions do not apply.

How well did you know this?
1
Not at all
2
3
4
5
Perfectly
72
Q

When is the Sixth Amendment violated in a joint jury trial?

A

In a joint jury trial, the Sixth Amendment confrontation clause is violated when a non-testifying co-defendant’s prior statement implicating the other defendant in the crime is admitted at trial.

How well did you know this?
1
Not at all
2
3
4
5
Perfectly
73
Q

Exceptions to the Parol Evidence Rule

A

Evidence of prior or contemporaneous oral or written agreement is admissible to establish:

1) whether writing is integrated and, if so, completely or partially
2) meaning of ambiguous term
3) defense to formation or enforcement (eg, fraud, duress, mistake)
4) ground for granting or denying remedy (eg, rescission, reformation)
5) subsequent contract modifications
6) condition precedent to effectiveness

How well did you know this?
1
Not at all
2
3
4
5
Perfectly
74
Q

First Amendment free exercise of religion

Level of Scrutiny

A

The First Amendment free exercise clause, applicable to the states through the Fourteenth Amendment, prohibits government interference with or discrimination against sincere religious beliefs or conduct. As a result, criminal prosecution under a statute that burdens a defendant’s religion can be challenged on these constitutional grounds.

However, the defendant’s likelihood of success depends on whether that burden is:

1) direct – the law intentionally targets religion and is usually unconstitutional under strict scrutinyor
2) incidental – the law generally applies to all and is usually constitutional under rational basis scrutiny.

How well did you know this?
1
Not at all
2
3
4
5
Perfectly
75
Q

Intrusion upon seclusion

A

Intrusion upon seclusion is an invasion of privacy that occurs when the defendant intentionally intrudes on the plaintiff’s private affairs in a manner that would be highly offensive to a reasonable person—eg, tapping telephone wires, examining private bank account, opening personal mail.

How well did you know this?
1
Not at all
2
3
4
5
Perfectly
76
Q

What defenses are waived if not included in a single pre-answer motion?

A

The following defenses are waived if they are not asserted in a single pre-answer motion or in an answer, whichever occurs first:

(1) lack of personal jurisdiction,
(2) improper venue, and
(3) insufficient process or service of process.

How well did you know this?
1
Not at all
2
3
4
5
Perfectly
77
Q

A contract is voidable for misrepresentation if:

A

A contract is voidable for misrepresentation if:

(1) the misrepresentation was fraudulent or material,
(2) it induced assent to the contract, and
(3) the adversely affected party justifiably relied on it.

How well did you know this?
1
Not at all
2
3
4
5
Perfectly
78
Q

When are liability waivers invalid?

A

A person can assume the risk of another’s tortious conduct by expressly waiving the right to sue. Such waivers are generally valid if the plaintiff assented (eg, by signing the waiver) AND the waiver was intended to cover the type of conduct that caused the plaintiff’s harm.

But liability waivers are invalid and cannot justify summary judgment if they are against public policy—ie, when the defendant:

(1) is the plaintiff’s employer,
(2) is a hotel or common carrier,
(3) is a public servant or service, or
(4) has substantially more bargaining power.

How well did you know this?
1
Not at all
2
3
4
5
Perfectly
79
Q

How can the recipient of a promissory note and mortgage lose the right of foreclosure?

A

The recipient of a promissory note and mortgage securing that note acquires the right of foreclosure.

However, the recipient can lose this right if the note and mortgage are subsequently conveyed to a bona fide purchaser—ie, one who pays value without notice of another’s prior interest.

How well did you know this?
1
Not at all
2
3
4
5
Perfectly
80
Q

The mortgagee is generally free to transfer the promissory note and/or the mortgage securing that note unless:

A

A mortgage is a document that gives the mortgagee (eg, bank) an interest in real property as security for an obligation owed by the mortgagor (ie, borrower). The obligation is typically memorialized in a promissory note.

The mortgagee is generally free to transfer the promissory note and/or the mortgage securing that note unless:

i) the mortgage or note expressly states otherwise
ii) the transfer is forbidden by statute or public policy OR
iii) the transfer will increase the duties, burdens, or risks on the mortgagor.

The recipient of the transferred note/mortgage (ie, transferee) then acquires the right of foreclosure.

  • However, the transferee can lose this right to a bona fide purchaser—ie, one who pays value for the property interest without notice of another’s prior interest in the property.
How well did you know this?
1
Not at all
2
3
4
5
Perfectly
81
Q

Adverse possession of a mineral estate

A

If the surface and mineral estates are owned by the same party, then the adverse possessor will acquire title to both estates—even if only one estate is actually possessed.

But if the mineral estate has been severed from the surface estate (ie, the surface and mineral estates are owned by different parties), then the adverse possessor will only acquire title to the estate that is actually possessed.

The mineral estate is actually possessed when the adverse possessor mines or drills wells on the land.

How well did you know this?
1
Not at all
2
3
4
5
Perfectly
82
Q

When is a mineral estate actually possessed for the purposes of adverse possession?

A

The mineral estate is actually possessed when the adverse possessor mines or drills wells on the land.

How well did you know this?
1
Not at all
2
3
4
5
Perfectly
83
Q

An appellate court will review a district court’s jury instructions under the following standards:

A

An appellate court will review a district court’s jury instructions under the following standards:

Abuse of discretion (high deference) – where the district court’s discretionary decisions (eg, the creation of jury instructions) will be reversed if they were unreasonable or arbitrary

De novo (no deference) – where a district court’s conclusions of law (eg, the contents of jury instructions) will be reversed if the appellate court reasonably believes that the court misinterpreted or misstated the law.

How well did you know this?
1
Not at all
2
3
4
5
Perfectly
84
Q

Risk of loss for specifically identified goods

A

Under the UCC, the risk of loss generally remains with the seller until the buyer receives the goods.

However, if the contract deals with specifically identified goods (eg, the antique car on display), then complete destruction of the goods excuses each party’s duty to perform so long as the destruction occurred:

i) without fault of either party and
ii) before the risk of loss passed to the buyer.

How well did you know this?
1
Not at all
2
3
4
5
Perfectly
85
Q

Doctrines affecting conveyance by will

A
  1. Lapse
    * Causes devise to fail if beneficiary predeceases testator
  2. Ademption
    * Causes devise to fail by either:
    1) extinction – specifically devised property not owned by testator (or destroyed or fundamentally changed) at death
    2) satisfaction – beneficiary received devised property (or other asset intended to satisfy devise) during testator’s life
  3. Exoneration
    * Allows beneficiary of specifically devised real property to use estate’s remaining assets to pay off any encumbrances on that property
  4. Abatement
  • Reduces devises that cannot be satisfied by assets remaining after testator’s debts are paid
  • Residuary devises abated first, followed by general & then specific devises
How well did you know this?
1
Not at all
2
3
4
5
Perfectly
86
Q

When can prior inconsistent statements be used to prove the truth of the matter asserted?

A

Prior inconsistent statements—ie, statements made by a witness in the past that are inconsistent with the witness’s current testimony—can be used to impeach a witness.

But these out-of-court statements can only be used to prove the truth of the matter asserted therein if they fall within an exclusion or exception to the hearsay rule.

How well did you know this?
1
Not at all
2
3
4
5
Perfectly
87
Q

To determine the amount of compensatory damages a plaintiff may recover in a negligence action, the following rules apply (unless the question otherwise indicates):

A
  1. Pure comparative negligence – when the plaintiff’s own negligence contributes to his/her harm, the plaintiff’s recovery is reduced by his/her proportionate share of fault
  2. Joint and several liability – when multiple defendants cause the plaintiff indivisible harm, the plaintiff can recover the total amount of damages from any defendant (who can then sue the others for contribution).
How well did you know this?
1
Not at all
2
3
4
5
Perfectly
88
Q

Venue refers to the federal judicial districts where a case may be heard and can be established in the following ways:

A
  1. Residency-based venue – a district where any defendant resides, if all of the defendants reside in the same state
  2. Events-based venue – a district where a substantial part of the events that gave rise to the suit occurred
  3. Property-based venue – a district where a substantial part of the property at issue is located (not seen here)
  4. Fallback provision – a district where any defendant is subject to the court’s personal jurisdiction (only applies if none of the above provisions can be established)

For venue purposes, a corporate defendant (eg, the shipping company) is a resident of any judicial district where it is subject to personal jurisdiction—eg, where it is incorporated or maintains its headquarters.

How well did you know this?
1
Not at all
2
3
4
5
Perfectly
89
Q

Regarding the best evidence rule, what issues does the jury determine?

A

Whether a party has fulfilled the factual conditions for admitting secondary evidence is ordinarily a determination for the court.

But in a jury trial, the jury determines any issue about whether:

1) an asserted document ever existed
2) another document produced at a hearing or trial is the original OR
3) other evidence of content accurately reflects the document’s content.

How well did you know this?
1
Not at all
2
3
4
5
Perfectly
90
Q

In slander cases, is the plaintiff’s character directly at issue?

A

Character evidence is generally inadmissible to prove that a person acted in conformity with that character on the occasion at issue. But such evidence is admissible when a person’s character is directly in issue—ie, when character is an essential element of a charge, claim, or defense. Character can then be proved by reputation or opinion testimony OR by specific instances of conduct.

In slander cases—where the defendant allegedly made a false statement that injured the plaintiff’s reputation—the plaintiff’s character is directly in issue for two reasons:

Truth is an absolute defense to slander, and the plaintiff’s actual character will determine whether the defendant’s statement was true.

The plaintiff will claim that he/she has been damaged by the defendant’s statement (ie, that his/her reputation has been tarnished), and damages are limited if the plaintiff already had a bad reputation.

How well did you know this?
1
Not at all
2
3
4
5
Perfectly
91
Q

Exception to the best evidence rule

A

The best evidence rule (ie, original document rule) applies when a witness relies on a document’s contents while testifying or when the contents of a document are at issue, such as when the document has a legal effect (eg, contracts).

This rule generally requires that an original or reliable duplicate of a recording, writing, or photograph(referred to as “document”) be produced to prove its contents. However, there are some exceptions.

One exception allows a party to introduce other evidence of the document’s content (eg, testimony) when the party against whom the original would be offered:

1) had control of the original
2) was put on notice (by pleadings or otherwise) that the original would be a subject of proof at trial and
3) failed to produce the original at trial.

How well did you know this?
1
Not at all
2
3
4
5
Perfectly
92
Q

Are drivers considered to be in custody for purposes of Miranda during traffic stops?

A

NO.

Miranda warnings must be provided to suspects before they are subjected to custodial interrogations. Since drivers are generally not in custody during traffic stops, Miranda warnings are not needed.

How well did you know this?
1
Not at all
2
3
4
5
Perfectly
93
Q

Attempt occurs when a defendant:

A

Attempt occurs when a defendant:

(1) has the specific intent to commit a crime,
(2) commits an overt act in furtherance of that crime, but
(3) does not complete it.

Under the substantial-step test used in the majority of jurisdictions, an overt act occurs when the defendant’s conduct exceeds mere preparation and strongly corroborates the defendant’s criminal intent.

Under the dangerous-proximity test (common law), Defendant performs act sufficiently close to completing crime—eg, act indispensable to crime’s success, act close in time or physical proximity to crime

How well did you know this?
1
Not at all
2
3
4
5
Perfectly
94
Q

Abandonment is not a defense to attempt if it was motivated by:

A

Abandonment is not a defense to attempt if it was motivated by:

(1) a desire to avoid detection,
(2) a decision to delay the commission until a more favorable time, or
(3) the selection of another similar objective or victim.

How well did you know this?
1
Not at all
2
3
4
5
Perfectly
95
Q

Is voluntary intoxication a defense to specific intent crimes like first-degree murder?

A

YES

Voluntary intoxication—deliberately ingesting a substance that a person knows or should know has intoxicating effects—is a defense to specific intent crimes (eg, first-degree murder) when the intoxication prevented the person from forming such intent.

How well did you know this?
1
Not at all
2
3
4
5
Perfectly
96
Q

Business Records Hearsay Exception

A

Under the hearsay rule, out-of-court statements (eg, entries in a hospital record) are generally inadmissible when offered to prove the truth of the matter asserted therein (eg, patient has two fractured vertebrae).

However, one hearsay exception applies to business records that were:

1) made at or near the time of the recorded event (or act, condition, opinion, diagnosis)
2) made by or based on information from someone with personal knowledge of that event AND
3) made and kept as a regular practice in the course of regularly conducted business activities.

How well did you know this?
1
Not at all
2
3
4
5
Perfectly
97
Q

Can a party immunize evidence from admission merely by producing it during settlement negotiations?

A

NO.

Statements made during settlement negotiations are generally inadmissible to prove/disprove the validity of a claim or impeach a party with a prior inconsistent statement. But a party cannot immunize evidence created for reasons other than settlement from admission merely by producing the evidence during settlement negotiations.

How well did you know this?
1
Not at all
2
3
4
5
Perfectly
98
Q

Battery committed in self-defense is justified if the person:

A

Battery occurs when a person inflicts harmful or offensive contact on another. But a battery committed in self-defense is justified—and not subject to criminal liability—if the person:

1) actually and reasonably believed that force was necessary to protect against imminent unlawful harm
2) used reasonable force (ie, no more than necessary) to prevent such harm AND
3) was not the initial aggressor—ie, did not provoke the altercation.

There is no duty to retreat before using nondeadlyforce in self-defense—even when one could do so safely.

But in the few jurisdictions that follow the retreat doctrine, a person must retreat before using deadly force when that person is outside his/her home and can retreat with reasonable safety.

How well did you know this?
1
Not at all
2
3
4
5
Perfectly
99
Q

Regarding class actions, when individual claims cannot be aggregated unless an alternate basis of SMJ over the class action exists, the Class Action Fairness Act (CAFA) provides such a basis when:

A

The Class Action Fairness Act gives a federal court an alternative basis for subject-matter jurisdiction over a class action when (1) the class contains 100 members, (2) at least one class member is diverse from at least one defendant, and (3) the amount in controversy of the aggregated claims exceeds $5 million.

A federal district court’s subject-matter jurisdiction over a class action, like any other lawsuit, can arise from either:

1) federal-question jurisdiction – the class action arises from the U.S. Constitution, a treaty, or federal law (not state law, as seen here) or
2) diversity jurisdiction – the named opposing parties are citizens of different states and the amount in controversyfor any named plaintiff’s claim exceeds $75,000.

Here, no single plaintiff can individually satisfy the amount-in-controversy requirement because the complaint alleges that the cell phone company overcharged each plaintiff only $70 ($3.50 × 20 months).

And individual claims generally cannot be aggregated (ie, added together) unless an alternate basis of subject-matter jurisdiction over the class action exists.

The Class Action Fairness Act (CAFA) provides such a basis when:

1) the class contains at least 100 members
2) any class member is diverse from at least one defendant (ie, minimal diversity) and
3) the amount in controversy exceeds $5 million when the members’ claims are aggregated.

How well did you know this?
1
Not at all
2
3
4
5
Perfectly
100
Q

Do federal courts have exclusive jurisdiction over class actions?

A

NO. Federal courts do not have exclusive jurisdiction over class actions.

The reason is that state and federal courts have concurrent jurisdiction over claims arising from state or federal law unless the Constitution or Congress expressly states otherwise.

How well did you know this?
1
Not at all
2
3
4
5
Perfectly
101
Q

Ways to discharge contractual obligations

A

Ways to discharge contractual obligations

Full performance of contractual obligations

Impossibility, impracticability, or frustration of purpose

Release (in writing only)

Mutual rescission

Substituted contract

Contract or covenant not to sue

Accord & satisfaction

Novation

Mnemonic: FIRM SCAN

How well did you know this?
1
Not at all
2
3
4
5
Perfectly
102
Q

The defense of impracticability is available when:

A

The defense of impracticability is available when:

1) an unanticipated event makes a party’s performance impracticable
2) nonoccurrence of the event was a basic assumption of the contract AND
3) the party seeking discharge is not at fault.

How well did you know this?
1
Not at all
2
3
4
5
Perfectly
103
Q

A conviction of larceny can stem from either:

A

Larceny occurs when there is a trespassory taking and carrying away of another’s personal property (actus reus) with the specific intent to permanently deprive that person of the property (mens rea).

A conviction can stem from either:

1) principal liability – when the actus reus is committed by the defendant OR a person acting on the defendant’s behalf without the requisite mens rea (ie, innocent agent) or
2) accomplice liability – when the defendant intentionally aids or encourages the principal before or during a crime with the intent that the crime be completed (making both parties equally liable for the crime).

How well did you know this?
1
Not at all
2
3
4
5
Perfectly
104
Q

A defendant can be criminally liable if he/she:

A

A defendant can be criminally liable if he/she:

(1) personally commits a crime or uses an innocent agent to do so (principal liability) or
(2) intentionally aids or encourages the principal before or during a crime with the intent that the crime be completed (accomplice liability).

But an accomplice is only liable to the same extent as the principal.

How well did you know this?
1
Not at all
2
3
4
5
Perfectly
105
Q

To prevail on a private nuisance claim, the plaintiff must prove that the defendant’s interference with the use and enjoyment of the plaintiff’s property was both:

A

To prevail on a private nuisance claim, the plaintiff must prove that the defendant’s interference with the use and enjoyment of the plaintiff’s property was both:

1) substantial – offensive, annoying, or intolerable to a normal person in the community AND
2) unreasonable – the gravity of the plaintiff’s harm outweighs the utility of the defendant’s conduct.

An interference that causes severe harm (eg, bodily harm, serious property damage) is unreasonable as amatter of law since severe harm always outweighs competing factors.

And once the plaintiff proves private nuisance, he/she can recover damages for that harm—including personal injuries.

How well did you know this?
1
Not at all
2
3
4
5
Perfectly
106
Q

Do class members in a class action certified as a “common question” class action have the right to opt out?

A

YES.

In “common question” class actions, adequate notice must be provided to all class members because they have the right to opt out of the class action and sue on their own behalf.*

If a class member fails to opt out, he/she cannot pursue an individual suit involving the same claim.

Instead, that class member is bound by any resulting final judgment or court-approved settlement, compromise, or voluntary dismissal.

*In “prejudicial risk” and “final equitable relief” class actions, notice is not required because class members do not have a right to opt out. Instead, a court has the discretion to provide notice and an opportunity to opt out of such class actions.

How well did you know this?
1
Not at all
2
3
4
5
Perfectly
107
Q

Which FRCP 12 defenses can be asserted later after a pre-answer motion?

A

Because of the omnibus motion rule, the following FRCP 12 defenses are waived and cannot be asserted later if omitted from a pre-answer motion:

(1) lack of personal jurisdiction, (2) improper venue, and (3) insufficient process or service of process.

However, the following FRCP 12 defenses may still be later asserted in an answer or post-answer motion if omitted from a pre-answer motion:

(1) failure to state a claim,
(2) failure to state a legal defense, and
(3) failure to join a required party.

How well did you know this?
1
Not at all
2
3
4
5
Perfectly
108
Q

When is misconduct prejudicial?

How can misconduct be made harmless?

A

A court can grant a new trial if a juror (or judge, party, or attorney) engaged in prejudicial misconduct during the trial. Misconduct is prejudicial when it affects a party’s substantial rights.

This requires the court to determine with fair assurance whether the jury’s verdict was influenced by the misconduct.

If there was no improper influence, then the misconduct was harmless and does not provide a basis for a new trial.

For example, misconduct can be made harmless (ie, “cured”) if the judge admonishes the jury to disregard that conduct.

How well did you know this?
1
Not at all
2
3
4
5
Perfectly
109
Q

Is a new trial warranted if a juror engages in prejudicial misconduct?

A

YES.

A new trial is warranted when a juror engaged in prejudicial misconduct—ie, misconduct that affects a party’s substantial rights by influencing the jury’s verdict.

If misconduct is not prejudicial, then it is merely harmless error that does not warrant a new trial.

How well did you know this?
1
Not at all
2
3
4
5
Perfectly
110
Q

Doctrine of Anticipatory Repudiation

When can a repudiation be retracted?

A

The doctrine of anticipatory repudiation applies when a contracting party clearly and unequivocally repudiates (ie, indicates an unwillingness to perform) a promise before the time for performance arises or elapses.

A repudiation can be retracted if the nonrepudiating party receives notice of the retraction before:

1) canceling the contract
2) materially changing position in reliance on the repudiation or
3) indicating that he/she considers the repudiation to be final.

If the repudiation is not retracted, the nonrepudiating party may (1) treat the repudiation as a breach or (2) ignore it and demand performance pursuant to the contract.

How well did you know this?
1
Not at all
2
3
4
5
Perfectly
111
Q

Is a person who finds lost or misplaced personal property guilty of larceny?

A

Yes, if a person who finds lost or misplaced personal property is guilty of larceny if he/she:

1) knows the identity of the owner or has reason to believe that he/she can determine it
2) takes and carries away the property—ie, exercises control over and moves it AND
3) possesses the specific intent to permanently deprive the owner of the property.

How well did you know this?
1
Not at all
2
3
4
5
Perfectly
112
Q

Plain View Doctrine

A

A court should suppress evidence that was obtained during an unreasonable Fourth Amendment search or seizure. A search or seizure is unreasonable when it is conducted without a warrant and no exception to the warrant requirement applies.

The plain view doctrine is an exception that allows an officer to seize an item without a warrant if:

1) the item is in the officer’s plain view
2) it is immediately recognized as contraband and
3) the officer is lawfully in the area.

An officer can lawfully enter a private area without a warrant if the officer obtains the owner’s consent.

  • Consent is a valid basis for entering an area if
    • (1) the owner’s consent is given voluntarily (ie, without police coercion) and
    • (2) any search is limited to the scope of the consent.
How well did you know this?
1
Not at all
2
3
4
5
Perfectly
113
Q

Exception to UCC Statute of Frauds for the sale of goods for $500 or more.

A

The UCC statute of frauds applies to contracts for the sale of goods for $500 or more (eg, private jet sold for $9 million).

These contracts generally must be in writing and signed by the party against whom enforcement is sought to be enforceable.

However, one exception applies when some amount of payment was made and accepted.* (this exception also applies when some amount of goods have been received and accepted.)

The effect of a partial payment turns on whether the contract is for:

1) separable goods – in which case, the contract is enforceable up to the quantity of goods paid for in full or
2) an indivisible good – in which case, the entire contract is enforceable.

How well did you know this?
1
Not at all
2
3
4
5
Perfectly
114
Q

What damages are available under the implied warranty of merchantability?

A

Under the UCC, contracts for the sale of goods come with an implied warranty of merchantability if the seller is a merchant with respect to the type of goods involved.

For example, under this warranty, damage caused by the seller’s failure to adequately package an item constitutes a breach.

If the buyer nevertheless chooses to accept the goods, the buyer’s damages are the difference between:

1) the value of the goods as accepted and
2) the value of the goods as warranted.

Repair costs are often used to determine this difference in value.

  • But if the repairs fail to restore the goods to their value as warranted, the buyer can recover the repair costs plus the difference between the value of the goods as warrantedand the value of the goods after the repairs.**
  • This assumes that the buyer paid for the repairs.
How well did you know this?
1
Not at all
2
3
4
5
Perfectly
115
Q

What are a buyer’s damages when repairs fail to restore goods to their value as warranted?

A

When repairs fail to restore goods to their value as warranted, the buyer’s damages are equal to the cost of repairs plus the difference between the value of the goods after the repairs and the value of the goods as warranted.

How well did you know this?
1
Not at all
2
3
4
5
Perfectly
116
Q

Defamation Liability

A

At common law, defamation liability could be strict. This means that the plaintiff was not required to prove that the defendant was at fault (eg, knew that the defamatory statement was false).

However, a number of Supreme Court decisions based on the First Amendment have since imposed the following fault requirements:

1) If the plaintiff is a public official or figure, the plaintiff must prove that the defendant acted with actual malice—ie, knowledge that the defamatory statement was false or reckless disregard for its falsity.
2) If the plaintiff is a private person and the defamatory statement involves a matter of public concern, the plaintiff must prove that the defendant was at least negligent as to the falsity of the statement.*

Example: Here, the man falsely told the reporter that the neighbor was dumping the contents of her septic tank into the lake. The neighbor is a private person and the man’s statement involves a matter of public concern—the potential pollution and destruction of the lake’s ecosystem. This means that the neighbor is constitutionally required to prove that the man was negligent. The neighbor can likely do so since the man made the statement to the reporter despite having no tangible proof that it was true. Therefore, the neighbor is likely to prevail

*If the plaintiff is a private person and the defamatory statement involves a matter of private concern, then the constitutional requirements do not apply. However, most states today require at least negligence by the defendant for all defamation actions.

How well did you know this?
1
Not at all
2
3
4
5
Perfectly
117
Q

If a party fails to preserve and challenge an error in a post-trial motion, what is the party’s only option for review on appeal?

A

A party must generally raise a formal objection at trial to preserve his/her right to challenge an error in a post-trial motion or on appeal. But if a party fails to do so, the challenged error can still be reviewed for plain error—ie, an obvious error that affected a substantial right and the fairness of judicial proceedings.

A party must generally raise a formal objection at trial to preserve and challenge an error in a post-trial motion (eg, motion for new trial) or on appeal.

An error in the jury instructions is preserved if the objection was raised either:

1) at the close of evidence – by filing a written request for a proposed instruction and obtaining a definitive ruling from the court on the record or
2) before the court instructed the jury and before closing arguments – by clearly identifying, and stating the grounds for, the objection on the record.

If a party failed to do so, the challenged error can only be reviewed for plain error.

Under a plain error review, a motion for new trial will be granted if the movant shows that an obvious error affected a substantial right and the fairness of judicial proceedings—eg, a jury instruction that misstated the law.

How well did you know this?
1
Not at all
2
3
4
5
Perfectly
118
Q

When are states not contrained by the dormant commerce clause?

A

The commerce clause grants Congress extensive authority to regulate interstate commerce. And the negative implication of this clause (ie, the dormant commerce clause) prohibits states from discriminating against or otherwise unduly burdening interstate commerce.

But states are not restrained by this clause when they participate in the market by, for example, buying or selling goods or services (ie, market-participant exception).

Example:

Here, the state discriminated against interstate commerce by refusing to supply electricity to out-of-state purchasers residing in states that would not store or dispose of spent fuel. But since the state itself owns and operates the power system, the market-participant exception applies. As a result, the state’s refusal to supply power is not subject to the negative implications of the commerce clause. Therefore, this is the strongest argument that its action is constitutional.

How well did you know this?
1
Not at all
2
3
4
5
Perfectly
119
Q

Can relevant evidence be barred by the best evidence rule?

A

Yes, if:

Relevant evidence is inadmissible if its probative value is substantially outweighed by its danger of unfair prejudice. And under the best evidence rule, relevant evidence in the form of a writing, recording, or photograph can be excluded if an original or reliable duplicate is not produced.

How well did you know this?
1
Not at all
2
3
4
5
Perfectly
120
Q

Is an earnest money down payment needed to form a valid land-sale contract?

A

NO. A buyer can make an earnest money down payment to show a good faith intent to complete the purchase, but this is not needed to form a valid contract.

  • Land-sale contracts are supported by valuable consideration because the seller promises to convey title to the buyer, and in exchange the buyer promises to pay the seller the purchase price.

A land-sale contract, like any other contract, is legally enforceable only if it is supported by valuable consideration. Valuable consideration is evidenced by a bargained-for exchange of legal positions between the parties.

This requires that each party:

1) perform, or promise to perform, some act that is not legally required OR
2) refrain, or promise to refrain, from performing some legally permissible act.

How well did you know this?
1
Not at all
2
3
4
5
Perfectly
121
Q

Statute of Frauds Requirements for real estate contracts

A

Statute of frauds requirements for real estate contracts

1) Writing
* Physical or electronic document(s) containing reasonable basis to believe contract was formed

eg, receipt, check, emails

2) Signature
* Handwritten signature from, or other mark identifying, party to be charged

eg, letterhead, electronic signature, initials

3) Essential terms

Typically includes:

  • identity of parties
  • words of intent to buy or sell
  • property description
  • sales price
How well did you know this?
1
Not at all
2
3
4
5
Perfectly
122
Q

Statute of Frauds Requirements for real estate contracts

Exception: Doctrine of Part Performance

A

The statute of frauds applies to all real estate contracts—including purchase and sale agreements between landlords and tenants in possession—and requires that the contract

(1) be in writing,
(2) be signed by the party against whom it is to be enforced, and
(3) contain all essential terms.

However, an oral agreement may still be enforceable if it falls within the doctrine of part performance. This exception to the statute of frauds applies when the buyer has done the following:

1) taken possession of the property (eg, the sister continued to live in the house)
2) made substantial improvements to the property (eg, the sister’s modification increased its value by 50%) and/or
3) paid some or all of the purchase price* (eg, the sister’s $25,000 cash payment and monthly payments)

The buyer can then enforce the oral agreement in equity, typically by seeking specific performance.

*Payment of some or all of the purchase price is not independently sufficient to establish part performance.

How well did you know this?
1
Not at all
2
3
4
5
Perfectly
123
Q

What are the requirements for a deed to properly transfer ownership?

A

A deed is a document that transfers ownership of real property from the owner (grantor) to another (grantee) once it has been:

1) delivered – presumed when the deed:
(1) has been recorded—ie, filed in the official land records—or
(2) is in the grantee’s physical possession AND
2) accepted – presumed when the transfer benefits the grantee.

Once these requirements are met, the grantee owns the property and the transfer cannot be canceled—even if the land record office later returns the deed to the grantor.

How well did you know this?
1
Not at all
2
3
4
5
Perfectly
124
Q

Actual Causation in wrongful death claims based on medical malpractice

Loss-of-chance doctrine

A
  • The loss-of-chance doctrine allows a plaintiff to recover in a wrongful death action based on medical malpractice if the defendant’s malpractice caused any reduction in the decedent’s chance of survival.*
    1) Traditional common law rule
  • Decedent probably would have survived (ie, greater than 50% chance) but for defendant’s malpractice
    2) Modern rule (loss-of-chance doctrine)
  • Decedent’s chance of survival was reduced by defendant’s malpractice (eg, 45% chance reduced to 20%)

Wrongful death statutes allow a decedent’s spouse, next of kin, or personal representative to sue a defendant for wrongfully causing the decedent’s death.

These lawsuits are often based on medical malpractice (eg, failure to diagnose), which requires the plaintiff to prove actual causation by a preponderance of the evidence.

Traditionally, this standard is met if it is more likely than not (ie, greater than 50% chance) that the decedent would have survivedbut for the defendant’s malpractice.

How well did you know this?
1
Not at all
2
3
4
5
Perfectly
125
Q

Is a person who unlawfully severs and then immediately carries away an item of real property guilty of common law larceny?

A

NO.

A person who unlawfully severs and then immediately carries away an item of real property—eg, a planted tree or fixture—is not guilty of common law larceny.

That is because such an item does not constitute personal property in its attached state.*

*In contrast, an item of real property that:

(1) was previously severed by the owner or
(2) was severed by the defendant and later came into the owner’s possession is personal property and subject to larceny.

How well did you know this?
1
Not at all
2
3
4
5
Perfectly
126
Q

Res Ipsa Loquitur

medical malpractice claims

A

1) Applicability

  • Plaintiff suffered unusual injury while unconscious during medical treatment
  • At least one member of medical team had control over patient’s body or instrumentality that caused injury

2) Effect

  • Creates inference that every member of medical team was negligent
  • Places burden on each member to establish that he/she was not negligent

Medical malpractice claims are based on negligence and generally require proof that the defendant violated a professional standard of care. But this can be difficult to prove when a complex procedure (eg, surgery) was performed by a team of medical professionals.

In that case, the doctrine of res ipsa loquitur lets the fact finder infer that everymember of the medical team was negligent when:

1) the patient suffered an unusual injury (ie, one that is usually caused by negligence) while unconscious and in the course of medical treatment AND
2) at least one member of the medical team had control over the patient’s body or instrumentalities that might have caused the patient’s injury.

How well did you know this?
1
Not at all
2
3
4
5
Perfectly
127
Q

An anti-lapse statute will save the gift if the recipient:

A

A gift conveyed under a will typically lapses if the recipient predeceases the testator.

However, an anti-lapse statute will save the gift if the recipient:

(1) has a specified family relation to the testator and
(2) is survived by his/her own children or other lineal descendants.

How well did you know this?
1
Not at all
2
3
4
5
Perfectly
128
Q

Sixth Amendment Right to a Jury Trial

Petition for a writ of habeas corpus

A

A petition for a writ of habeas corpus is generally used as a post-conviction remedy to collaterally attack an unlawful imprisonment. If the petitioner is incarcerated for a state conviction, the petition can only be filed in federal court if the petitioner has exhausted all state appellate remedies. The federal court will only grant the petition and set aside the conviction if the petitioner’s incarceration violates the U.S. Constitution.

The Sixth Amendment right to a jury trial attaches to any criminal proceeding where the defendant could be sentenced to more than six months in prison.

This right, applied to the states through the Fourteenth Amendment, requires that a jury

(1) have at least six members and

(2) reach a unanimous verdict to convict a defendant. If these requirements are not met, any conviction and subsequent incarceration are unconstitutional.

How well did you know this?
1
Not at all
2
3
4
5
Perfectly
129
Q

A contract is voidable for undue influence if:

A

A contract is voidable for undue influence if a party’s assent was the product of unfair or excessive persuasion by someone who dominated or shared a special relationship of trust and confidence with that party.

Persuasion is unfair or excessive if it seriously impairs the assenting party’s ability to exercise free and competent judgment.

The following factors are relevant to making this determination:

1) Susceptibility of the assenting party (eg, elderly, in poor health)
2) Unfairness of the resulting bargain (eg, sale of business for less than half its market value)
3) Unavailability of independent advice (eg, attorney unavailable to discuss sale)

How well did you know this?
1
Not at all
2
3
4
5
Perfectly
130
Q

A contract is voidable for a misrepresentation that:

A

A contract is voidable for a misrepresentation that

(1) was fraudulent or material,
(2) induced assent to the contract, and
(3) was justifiably relied on by the assenting party.

How well did you know this?
1
Not at all
2
3
4
5
Perfectly
131
Q

Profit

Right to enter another’s land & remove natural resources

A

1) Exclusive
* Characteristics*

Unlimited & exclusive right

Transferability

Can be transferred unless profit is personal or contrary to parties’ intent

Apportionability

If transferable, can be divided unless contrary to parties’ intent or burden on servient estate unreasonably increased

2) Non-exclusive
* Characteristics*

Right limited by quantity, use, or time OR shared with another

Transferability

Can be transferred unless profit is personal or contrary to parties’ intent

Apportionability

If transferable, can be divided unless contrary to parties’ intent or burden on servient estate unreasonably increased

How well did you know this?
1
Not at all
2
3
4
5
Perfectly
132
Q

When does the “one-stock” rule apply?

A

When a profit is apportioned or divided up among multiple transferees, the “one stock” rule applies.

Under this rule, the transferees are limited to the amount of material taken by the transferor (ie, the transferor’s “stock”), and this quantity is divided up by the transferees taking the profit.

This ensures that the burden on the servient (burdened) estate is not unreasonably increased.

How well did you know this?
1
Not at all
2
3
4
5
Perfectly
133
Q

Good faith exception to exclusionary rule

A

The exclusionary rule seeks to deter police misconduct by prohibiting the government from using evidence obtained in violation of a defendant’s constitutional rights at trial.

But under the good faith exception, illegally obtained evidence is admissible if the prosecution proves by a preponderance of the evidence that:

i) police relied in good faith on (1) a facially valid warrant later deemed invalid or (2) an existing law later declared unconstitutional AND
ii) their reliance was objectively reasonable.

How well did you know this?
1
Not at all
2
3
4
5
Perfectly
134
Q

Ways to add parties

A

1) Required joinder
* Requires addition of necessary party to suit
2) Permissive joinder
* Allows addition of nonessential party to suit
3) Intervention
* Allows nonparty whose interests may be affected to join suit
4) Impleader
* Allows defendant to add nonparty who may be liable to defendant for all or part of asserted claim to suit
5) Interpleader
* Allows possessor of property to force persons who claim ownership of property to resolve dispute in single suit
6) Class action
* Allows party to represent interests of entire class of similarly situated individuals

How well did you know this?
1
Not at all
2
3
4
5
Perfectly
135
Q

Impleader (i.e., third-party practice)

A

Impleader (ie, third-party practice) allows a defendant to add a nonparty to a suit who may be liable to the defendantfor all or part of the plaintiff’s claim.

This means that the nonparty would reimburse the defendant for any damages paid to the plaintiff.

To initiate impleader, the defendant can file a third-party complaint:

(1) within 14 days of servinghis/her original answer OR
(2) after this deadline with the court’s permission.

How well did you know this?
1
Not at all
2
3
4
5
Perfectly
136
Q

What limited duty does a landowner owe an invitee who becomes a known or anticipated trespasser?

A

Under the traditional common law approach,* land possessors owe a duty of reasonable care to foreseeable land entrants—including invitees. But an invitee is treated as a trespasser if that person intentionally enters an area without permission.

And if that person is a known or anticipated trespasser, the landowner owes a limited duty to:

1) warn the trespasser about hidden, artificial (ie, man-made) dangers that are known to the land possessor but unlikely to be discovered by the trespasser AND
2) use reasonable care in active operations (ie, activities conducted on the land).

A land possessor who breaches this duty and causes the trespasser physical harm is liable for negligence. And if the land possessor is an employer, the doctrine of respondeat superior holds the land possessor vicariously liable for its employees’ negligence while on the job.

How well did you know this?
1
Not at all
2
3
4
5
Perfectly
137
Q

Negligence

To prevail on a negligence claim, the plaintiff must prove the following elements:

A

Negligence is the typical claim for unintended personal injuries.

To prevail on a negligence claim, the plaintiff must prove the following elements:

1) Duty – the defendant owed a duty of reasonable care because his/her conduct created a foreseeable risk of harm to others
2) Breach – the defendant breached that duty by failing to use reasonable care
3) Causation – the plaintiff’s harm would not have occurred absent the defendant’s breach (factual cause) and that type of harm was foreseeable (proximate cause)
4) Damages – the plaintiff suffered physical harm (ie, bodily harm or property damage).

A defendant fails to use reasonable care when he/she does not take reasonable precautions to reduce the risk posed by his/her conduct.

How well did you know this?
1
Not at all
2
3
4
5
Perfectly
138
Q

Voiding / ratifying contract by minor

A

Unmarried, unemancipated minors (ie, persons under the age of 18) lack capacity to contract because of their infancy. As a result, any contract entered by a minor is voidable by the minor.*

This means that the minor can either:

i) *void* the contract before or soon after turning 18 by:
(1) disaffirming the contract through words or actions and
(2) returning any still-existing benefit received under the contract OR
ii) *ratify* the contract after turning 18 by:
(1) failing to timely void the contract or
(2) otherwise agreeing to be boundby it—eg, by attempting to render performance of or payment under the contract.

The contract becomes enforceable if the minor ratifies it.

How well did you know this?
1
Not at all
2
3
4
5
Perfectly
139
Q

A prosecution for attempted murder requires proof that the defendant:

A

A prosecution for attempted murder requires proof that the defendant:

1) had the specific intent to commit murder
2) committed an overt act in furtherance of that crime but
3) did not complete it.

And attempt, like any other crime, also requires proof of concurrence between the defendant’s mental state and criminal act.

Therefore, the victim of an attempted murder is the intended target of the defendant’s criminal act (attempted killing) committed with the requisite mental state (specific intent).

And though a defendant’s intent to harm one person may be transferred to another unintentionally harmed by the defendant, the doctrine of transferred intent does not apply to attempt.

How well did you know this?
1
Not at all
2
3
4
5
Perfectly
140
Q

Does the doctrine of transferred intent apply to attempt?

A

NO. The doctrine of transferred intent does not apply to attempt.

How well did you know this?
1
Not at all
2
3
4
5
Perfectly
141
Q

In most jurisdictions, burglary is defined as the:

A

In most jurisdictions, burglary is defined as the:

1) unlawful entry of a building or other structure
2) with the specific intent to commit any crime therein—even if that crime is not completed.
* An unlawful entry occurs when any part of the defendant’s body (or an object under his/her control) crosses into the structure without the possessor’s consent or legal privilege.

And the crime that burglars often intend to commit inside is larceny—ie, the unlawful taking and carrying away of another’s personal property with the specific intent to permanently deprive that person of the property.

How well did you know this?
1
Not at all
2
3
4
5
Perfectly
142
Q

What is the “negative implications of the commerce clause” also known as?

A

The Dormant Commerce Clause

Under the negative implications of the commerce clause (ie, the dormant commerce clause), states are generally prohibited from unduly burdening interstate commerce.

  • An undue burden can arise when a state discriminates against interstate commerce by favoring in-state over out-of-state economic interests. But such discrimination is permitted when Congress explicitly consents to the state’s regulation.
How well did you know this?
1
Not at all
2
3
4
5
Perfectly
143
Q

When would a state law that would ordinarily violate the dormant commerce clause be valid?

A

A state law that would ordinarily violate the dormant commerce clause is valid if an exception exists—eg, when Congress explicitly consents to such a law.

How well did you know this?
1
Not at all
2
3
4
5
Perfectly
144
Q

Assignee-landlord’s right to enforce lease covenant that runs with land

A

A landlord may generally transfer his/her property interest to a third party (ie, assignee-landlord) without the tenant’s consent. However, the assignee-landlord only has the right to enforce a covenant (ie, promise) in the lease if it runs with the land.

Otherwise, the original landlord retains the right to enforce it. A covenant runs with the land when:

1) the original parties intended to bind successors in interest (eg, assignee-landlord)
2) the covenant touches and concerns (ie, affects the use or value of) the land AND
3) the transfer brings the assignee-landlord into privity of estate (ie, a mutual or successive relationship in the same property interest) with the tenant.

How well did you know this?
1
Not at all
2
3
4
5
Perfectly
145
Q

What is the only way a lease covenant can be enforced by an assignee-landlord?

A

A lease covenant can only be enforced by an assignee-landlord if it runs with the land.

This occurs when:

(1) the original parties intended to bind their successors,
(2) the covenant touches and concerns the land, and
(3) there is privity of estate.

How well did you know this?
1
Not at all
2
3
4
5
Perfectly
146
Q

Voir Dire

A

Voir dire is the stage in a jury trial when the court, parties, or attorneys question potential jurors about their backgrounds and potential biases. During this process, each party may strike potential jurors from serving on the jury through:

1) three peremptory challenges, which can be used to strike a potential juror for any reason (other than race, ethnicity, or gender) without explanation AND
2) unlimited challenges for cause, which can be used to strike a potential juror for bias or lack of impartiality.

How well did you know this?
1
Not at all
2
3
4
5
Perfectly
147
Q

What are the four theories of invasion of privacy?

When does liability for appropriation of name or likeness arise?

A

There are four theories of invasion of privacy:

(1) appropriation of name or likeness,
(2) intrusion upon seclusion,
(3) publicity given to private facts, and
(4) publicity placing person in false light.

Liability for appropriation of name or likeness (ie, misappropriation of the right to publicity) arises when a defendant:

1) uses the plaintiff’s name, picture, or likeness without authorization AND
2) obtains a benefit (typically a commercial advantage—eg, using the plaintiff’s picture to advertise a product).

How well did you know this?
1
Not at all
2
3
4
5
Perfectly
148
Q

What Amendment bars states from enacting regulations that unduly burden a woman’s fundamental right to obtain an abortion prior to fetal viability?

A

The Fourteenth Amendment due process clause bars states from enacting regulations that unduly burden a woman’s fundamental right to obtain an abortion prior to fetal viability.

But the Supreme Court has repeatedly held that state laws requiring parental notice or consent for minors (under the age of 18) are permissible burdens if they include a judicial-bypass procedure that meets all the following criteria:

1) It allows the minor to show that (1) she has the necessary maturity and information to make her own decision regarding abortion or (2) the abortion would be in her best interest.
2) It ensures the minor’s anonymity.
3) It is expedient enough to give the minor an effective opportunity to obtain an abortion.

How well did you know this?
1
Not at all
2
3
4
5
Perfectly
149
Q

What is required for parental-notice-or-consent laws regarding a minor’s due process right to an abortion?

A

Parental-notice-or-consent laws respect a minor’s due process right to an abortion if the law has a judicial-bypass procedure that:

(1) allows the minor to show that she is capable of making this decision or the abortion is in her best interest,
(2) ensures the minor’s anonymity, and
(3) gives the minor an effective opportunity to obtain an abortion.

150
Q

Can a state court remove a case to federal court on its own initiative?

A

NO.

A state court cannot remove a case to federal court on its own initiative—even if the suit involves a substantial federal question. Instead, the defendant must request removal.

151
Q

Should the courts defer to Congress when determining whether federal spending advances the general welfare?

A

YES.

The taxing and spending clause gives Congress extensive power to spend for the general welfare (ie, any public purpose).

And the Supreme Court has indicated that courts should defer to Congress when determining whether an instance of federal spending advances the general welfare.

152
Q

When may a federal district court impose reasonable sanctions for noncompliance with pretrial conference or order?

A

A federal district court may order the attorneys and any unrepresented parties to attend one or more pretrial conferences. These conferences are designed to improve the trial’s quality, facilitate settlement, and expedite litigation (eg, by obtaining admissions and stipulations).

To promote those goals, a district court may impose reasonable sanctions on any party or attorney who:

1) failed to attend a pretrial conference
2) was substantially unprepared to participate in the conference
3) did not participate in good faith in the conference or
4) failed to obey a pretrial order.

Additionally, a court must order a noncompliant party and/or attorney to pay the other party’s reasonable expenses, including attorney’s fees, unless:

(1) the noncompliance was substantially justified OR
(2) the payment would be unjust.

153
Q

Do landowners have a right to lateral support from all adjoining land?

A

YES.

Landowners have a right to lateral support from all adjoining land.

Therefore, an adjoining landowner who withdraws lateral support for land in a natural state is strictly liable for any subsidence (ie, cave-in, slippage) that occurs because of that withdrawal.

But if the adjoining landowner withdraws lateral support for improved land—as seen with the man’s industrial building—the adjoining landowner may be:

1) strictly liable if the landowner can show that damage to the land and improvements would have occurred in the land’s natural state OR
2) liable for negligence if the landowner cannot make such a showing.

Example from question:

Here, the friend’s excavation was not done negligently or with any malicious intent to injure the man. This means that the man can only recover damages resulting from the injuries to his improved land if the friend is subject to strict liability. And if the man shows that the subsidence would have occurred without the weight of the building on his lot, he will likely prevail.

154
Q

What is the “bursting bubble” approach to presumptions in civil cases?

A

Under the “bursting bubble” approach* followed by the FRE in civil cases, a party can overcome a presumption (eg, that neither spouse survived the other) by producing sufficient contrary evidence.

  • Once this occurs, the presumption “bursts” and no longer governs the matter.
  • *This bursting-bubble approach does not apply to conclusive presumptions or in federal diversity cases, where state law governs the effect of a presumption.*
155
Q

When is a commercial seller in the distribution chain subject to strict products liability?

A

Any commercial seller in the distribution chain (eg, manufacturer, distributor, retailer) is subject to strict products liability if:

(1) the commercial seller’s product was defective when it left the commercial seller’s control AND
(2) that defect caused the plaintiff harm.

Strict liability is imposed even if the commercial seller did not create or know about that defect.

156
Q

Can photographs constitute defamation as libel?

A defamation claim based on libel ordinarily requires proof that:

A

YES.

Defamation can be either slander (spoken statements) or libel (statements made in writing, photographs, or other physical forms—as seen here).

A defamation claim based on libel ordinarily requires proof that:

1) the defendant knowingly made a false statement about the plaintiff or negligently failed to determine its falsity
2) that type of statement would tend to harm the plaintiff’s reputation AND
3) the defendant intentionally or negligently communicated that statement to a third party.

But when the plaintiff is a public figure or official (eg, celebrity, governor), the false statement must be made with actual malice—ie, the defendant knew that the statement was false or recklessly disregarded its falsity.

Proof that the defendant negligently failed to determine the statement’s falsity is insufficient.

157
Q

To recover for defamation, a public figure or public official must prove that the defendant:

A

To recover for defamation, a public figure or public official must prove that the defendant:

1) acted with actual malice—ie, with knowledge of (or reckless disregard for) the falsity of his/her defamatory statement.

158
Q

The commerce clause gives Congress broad power to regulate interstate and foreign commerce.

This includes:

A

The commerce clause gives Congress broad power to regulate interstate and foreign commerce. This includes:

1) the channels of interstate and foreign commerce (eg, roadways)
2) the instrumentalities of interstate and foreign commerce (eg, vehicles)
3) persons and things moving in interstate or foreign commerce (eg, goods and services) AND
4) in-state activities that, singly or in the aggregate, substantially impact interstate or foreign commerce.

159
Q

Does Congress’s regulation of prices constitute taxing or spending?

A

NO.

The taxing and spending clause empowers Congress to tax and spend for the general welfare.

  • But regulating prices is not equivalent to taxing or spending.
160
Q

In a joint jury trial—ie, when two or more defendants are tried before the same jury on charges related to the same facts— when is a defendant’s right to confrontation is violated?

A

A court should exclude testimony that would violate a criminal defendant’s constitutional rights, including the Sixth Amendment right to confront (ie, cross-examine) adverse witnesses at trial.

In a joint jury trial—ie, when two or more defendants are tried before the same jury on charges related to the same facts—a defendant’s right to confrontation is violated when:

1) the court admits a co-defendant’s out-of-court statement that implicates the other defendant in the crime AND
2) the co-defendant does not testify at trial and cannot be compelled to testify due to his/her Fifth Amendment privilege against self-incrimination.

A limiting instruction cannot cure this violation.

161
Q

What 3 Bill of Rights do not apply to the states?

A

The first 10 amendments to the U.S. Constitution (ie, the Bill of Rights) protect individuals from government overreach.

These protections limit the power of the federal government, and most apply to the states with equal force through incorporation into the Fourteenth Amendment due process clause—with three exceptions:

1) The Third Amendment prohibition against the government quartering soldiers in individuals’ homes
2) The Fifth Amendment requirement of a grand jury indictment to charge a person with a capital offense
3) The Seventh Amendment guarantee of a jury trial in civil cases*

162
Q

How is an easement by prescription acquired?

A

An easement is a nonpossessory interest in land that gives the easement holder the right to use another’s land for a limited purpose (eg, to use the path).

One type of easement is an easement by prescription, which is acquired through use that is:

1) Open and notorious – apparent or visible to a reasonable owner
2) Continuous – uninterrupted for the statutory period
3) Actual – use of the land
4) Nonpermissive – hostile and adverse to the owner (ie, without the owner’s permission)

Unlike with adverse possession, the use need not be exclusive of the owner or other users—including the owner’s tenants (eg, the hunters).

163
Q

A law that discriminates based on gender or illegitimate birth (ie, quasi-suspect classes) must survice what level of scrutiny?

A

A government action that treats similarly situated persons differently may be challenged under the equal protection clause.

The action will only be deemed constitutional if there is adequate justification for the alleged discrimination.

A law that discriminates based on gender or illegitimate birth (ie, quasi-suspect classes) is presumptively invalid unless it survives intermediate scrutiny—ie, the government demonstrates that the law is substantially related to an important government interest.

Example: Here, a public high school offers child-care classes to students with children (government action). But enrollment is limited to female students with children in the school’s infant day-care center (gender discrimination). And since the male student was excluded from these classes, the admission policy will only be found constitutional if the school meets its burden of persuasion under intermediate scrutiny.

164
Q

Sanction proceedings can be initiated:

A

Sanction proceedings can be initiated:

1) on the court’s own initiative (ie, sua sponte) – which requires the judge to issue an order to show cause to the alleged violator or
2) by a party’s motion – which requires the party to serve the motion on the alleged violator but refrain from filingit for 21 days after serving it to allow any violation to be corrected (ie, safe-harbor rule).

165
Q

Covenant of Warranty

A

A warranty deed conveys a grantor’s real property to a grantee with assurances that there are no defects in title—ie, no one else owns the land and it is free from encumbrances.

The covenant of warranty is an assurance that the grantor will defend and compensate the grantee for lawful claims made against the grantee’s title—eg, when a third party prevails against the grantee in a quiet title action.

166
Q

Under the covenant of warranty, when must the grantor of the warranty deed defend and compensate the grantee for any lawful claims against the grantee’s title?

A

Under the covenant of warranty, the grantor of a warranty deed must defend and compensate the grantee for any lawful claims made against the grantee’s title—eg, when the grantee is defeated in a third party’s quiet title action.

167
Q

Can a defendant be convicted for felony murder if they do not have the intent to commit the underlying felony?

A

NO. Felony murder requires proof that:

(1) the defendant intended to commit an inherently dangerous felony AND
(2) a killing occurred during the commission or attempted commission of that felony—regardless of whether the death was caused by the actions of the defendant or a co-felon.

In most jurisdictions, felony murder is a killing that occurs during the commission, or attempted commission, of an inherently dangerous felony.

A defendant can be convicted if the death was caused by his/her own actions or a co-felon’s actions—even if the killing was accidental or occurred without the defendant’s knowledge.

But the defendant must have had the required mental state for felony murder—the intent to commit the underlying felony. For robbery, this requires the intent to steal another’s property.

168
Q

Perfect Tender Rule

Installment Contracts

A

In an installment contract for the sale of goods, the buyer must accept a nonconforming shipment if the seller gives adequate assurance that he/she will cure the defect.

And the buyer cannot cancel the rest of the contract unless the nonconforming shipment substantially impairs the value of the whole contract.

169
Q

Can a plaintiff recover compensatory damages for unusual or unforseeable harm under a negligence claim?

A

YES.

A successful negligence claim requires proof that the plaintiff suffered physical harm (ie, personal injury or property damage) as a result of the defendant’s negligent conduct.

Assuming all other elements are proved, the plaintiff can then recover compensatory damages for his/her initial physical harm AND any subsequent harm (physical, economic, or emotional) that is traceable to that initial harm.

This is true even if the extent of the harm was unusual or unforeseeable because “the defendant takes the victim as found.”*

  • This is known colloquially as the “thin-skull” or “eggshell plaintiff” rule.
170
Q

A lay witness can generally testify to any relevant matter based on his/her personal knowledge.

This includes both:

A

A lay witness can generally testify to any relevant matter based on his/her personal knowledge.

This includes both:

1) factual matters that the witness perceived firsthand—eg, “I saw a man holding a beer bottle and slurring his words” AND
2) common-sense impressions that are rationally based on those perceptions—eg, “The man was drunk.”

The best evidence rule generally requires that an original writing, recording, or photograph (eg, the ledger) be admitted to prove its contents. (e.g.,–But this rule does not affect the admissibility of the defendant’s testimony to rebut the ledger’s contents.)

171
Q

When is demonstrative evidence inadmissible?

A

Demonstrative evidence includes videos, photographs, models, and other devices used to explain a fact at issue in a case.

The admissibility of such evidence is governed by the balancing test set forth in FRE 403.

  • Under this rule, a demonstration is inadmissible if its probative value is substantially outweighed by the danger of unfair prejudice, confusion, misleading the jury, undue delay, wasting time, or needless cumulation.
172
Q

Writ of Mandamus

A

Absent limited exceptions, a party cannot seek appellate review of a district court’s order until a final judgment has been entered.

A petition for a writ of mandamus is one such exception.

It requests that an appellate court direct a district court to do or refrain from doing something when exceptional circumstances clearly exist and no other adequate remedy is available.

As a result, this writ is appropriate only in extremely limited situations—eg:

1) when a district court clearly exercises unlawful jurisdiction
2) when a district court clearly usurps another branch’s power and threatens separation of powers
3) to protect a party’s Seventh Amendment right to a jury trial

173
Q

What is the Youngstown Framework when determining whether the President acted within his/her constitutional authority?

A

Article II grants the President constitutional authority over a wide range of domestic and foreign policy areas, including serving as commander in chief of the U.S. military. Congress also has authority over military matters, including the power to establish military courts and tribunals.

When these coequal branches of government share power, the Youngstown framework determines whether the President acted within his/her constitutional authority.

This framework considers whether there was congressional:

1) authorization – in which case the President had maximum authority to act and that act is likely valid
2) silence – in which case the act is valid so long as it (1) fell within the President’s Article II powers and (2) did not interfere with another branch’s powers or
3) opposition – in which case the President had minimal authority to act and that act is likely invalid.

174
Q

When can a misrepresentation be cured, making the contract no longer voidable?

A

A fraudulent or material misrepresentation can be cured—and the contract is no longer voidable by the adversely affected party—if the circumstances change in such a way that the misrepresentation becomes true before the adversely affected party avoids the contract.

175
Q

Affiliates to the federal government are subject to state regulation unless:

A

Pursuant to the supremacy clause, the federal government is immune from state regulation without congressional consent.

But affiliates of the federal government—ie, persons or entities employed by or doing business with the federal government (as seen here)—are subject to state regulation unless:

1) Congress grants the affiliate immunity
2) the regulation discriminates against the federal government or its affiliates OR
3) the regulation substantially interferes with the affiliate’s federal purpose or duties.

176
Q

Hearsay Exception

Statement concerning the declarant’s then-existing state of mind or emotional/physical condition

A

Statements regarding a declarant’s then-existing state of mind (eg, motive, intent, plan) or emotional/physical condition (eg, mental feeling, pain, bodily health) are excepted from the rule against hearsay.

177
Q

Then-existing state of mind hearsay exception

A

Type of statement

1) State of mind

- Examples: Motive, intent, plan

- Admissibility: Admissible as substantive evidence

2) Emotional or physical condition

- Examples: Fear, pain, bodily health

- Admissibility: Admissible as substantive evidence

3) Memory or belief

- Examples: Reflection on prior event or belief about matter

- Admissibility: Inadmissible unless related to declarant’s will

178
Q

What future interest follows a life estate?

A

A life estate is a present possessory interest in real property that expires upon the death of an individual.

The future interest that follows a life estate is either a reversion (if it is held by the grantor) or a remainder (if it is created in a grantee).

A remainder is contingent if it is held by an unknown or unborn person OR subject to a condition precedent.

179
Q

Remainders are contingent if they are:

A

Future interests following a life estate are called remainders if they are created in a grantee.

And remainders are contingent if they are:

(1) held by an unknown or unborn person or
(2) subject to a condition precedent.

180
Q

Present Sense Impression Hearsay Exception

Impeaching a witness using SICs

A

A statement describing an event or condition made while or immediately after the declarant perceived it.

When an out-of-court statement is admitted under a hearsay exception, the declarant can be impeached as if the declarant had testified at trial—eg, with a bad act that is probative of the declarant’s character for (un)truthfulness.

When a hearsay statement is admitted, the hearsay declarant can be impeached as if he/she were a witness testifying at trial.

One method of impeaching a witness is by introducing evidence of specific instances of conduct (SICs) that are probative of the witness’s character for (un)truthfulness.

Only two types of SICs are admissible for this purpose:

1) convictions for a felony or crime of dishonesty and
2) other bad acts.

181
Q

Is mistake of fact a defense to burglary (and other specific intent crimes)?

A

YES.

A mistake of fact is a defense to burglary (and other specific intent crimes) when the defendant’s honest but mistaken belief negates the requisite intent.

This is true even when that mistake is unreasonable.

182
Q

Examples of contracts unenforceable on public-policy grounds:

A

Common examples:

Contracts unreasonably restraining trade

Contracts discouraging or restraining marriage

Contracts for sexual intercourse

Contracts for committing a tort

Contracts encouraging breach of a fiduciary duty

Contracts excusing liability for intentional, knowing, or reckless misconduct

183
Q

When a contract violates a law that was designed to protect the party seeking to enforce the contract, can the contract be enforced?

A

YES.

When a contract violates a law that was designed to protect the party seeking to enforce the contract, the contract can be enforced to avoid frustrating the public policy behind the statute.

184
Q

What are the grantee’s obligations to a mortgage when they have assumed the mortgage?

A

A mortgage is an interest in real property given to secure a debt.

The debtor may freely transfer mortgaged property (unless otherwise agreed), but the mortgage will remain attached to that property even after it is transferred to another.

The grantee’s (eg, buyer’s) obligations with respect to that mortgage depend on whether he/she either:

1) took subject to the mortgage – did not agree to pay and is not personally liable for the debt OR
2) assumed the mortgage – expressly agreed to pay and becomes primarily liable for the debt, while the original debtor becomes secondarily liable as a surety.

185
Q

Does due process regulate affirmative defenses?

A

NO.

Due process does not apply to affirmative defenses (eg, insanity), so state law can require the defendant to prove an affirmative defense by any standard. Additionally, a legislature can freely modify or eliminate these defenses.

186
Q

Does a court typically have personal jurisdiction in an in rem action?

A

A court almost always has personal jurisdiction in an in rem action (eg, civil forfeiture action).

Minimum contacts exist because an in rem action relates to property in the forum state.

And exercising jurisdiction complies with fair play and substantial justice so long as the plaintiff did not fraudulently bring the property into the forum state.

187
Q

When is an expert’s testimony admissible for substantive purposes?

A

Expert testimony is admissible as substantive evidence if it will help the trier of fact understand the evidence or determine a fact in issue (ie, relevant) AND is based on sufficient facts/data and the product of reliable principles/methods reliably applied by the expert (ie, reliable).

An expert witness is one who possesses specialized knowledge, skill, experience, education, or training in a subject that pertains to an issue in litigation.

Expert witness testimony is admissible for substantive purposes—ie, to help prove a material fact or issue—if it is both:

i) relevant – the testimony will help the trier of fact:
(1) understand the evidence OR
(2) determine a fact in issue AND
ii) reliable – the testimony is:
(1) based on sufficient facts or data AND
(2) the product of reliable principles and methods that the expert reliably applied to the facts of the case.

188
Q

Are claims for purely economic loss allowed in strict products liability cases?

A

NO.

A strict products liability claim requires proof of the following elements:

1) The defendant(s) were commercial sellers (eg, manufacturer, retailer—).
2) The product was defective at the time it left the defendant’s control.
3) The defect caused the plaintiff physical harm (ie, bodily harm or property damage).

As a result, claims for purely economic loss are not allowed.

  • In products liability cases, purely economic loss includes:
    • any harm to the defective product itself as well as consequential damages arising therefrom (eg, lost profits).

NOTE:

  • However, a products-liability action brought under warranty theories can recover for purely economic loss.

​Any product that fails to live up to either of the above warranties constitutes a breach of the defendant’s warranty; the plaintiff need not prove any fault on the defendant’s part.

The buyer may recover damages for personal injury and property damage, as well as for purely economic loss.

189
Q

When can the adversely affected party void a contract due to mutual mistake?

A

Mutual mistake occurs when both parties enter into a contract based on the same mistake.

In such cases, the contract may be voidable by the adversely affected party if:

1) the mistake relates to a basic assumption of the contract
2) the mistake materially affects the agreed-upon exchange of performances (ie, the adversely affected party cannot fairly be required to carry it out) AND
3) the adversely affected party did not assume the risk of the mistake.

190
Q

When a tenant attaches tangible personal property (ie, a chattel) to the leased premises, the tenant can remove the chattel before, or within a reasonable time after, the lease expires if the chattel is not a fixture—ie, if:

A

When a tenant attaches tangible personal property (ie, a chattel) to the leased premises, the tenant can remove the chattel before, or within a reasonable time after, the lease expires if the chattel is not a fixture—ie, if:

1) the attachment of the chattel was not intended to be permanent OR
2) the chattel is not being used for some larger component (eg, lighting, water drainage) or function (eg, plumbing, septic) of the land.

And when a landlord conveys leased premises subject to an existing lease, the terms of the lease govern the new landlord-tenant relationship.

191
Q

Can an expert witness’s opinion be based on a hearsay statement?

A

Yes, if other experts in the field would reasonably rely on that statement to form an opinion.

Hearsay is an out-of-court statement (eg, the doctor’s letter) offered to prove the truth of the matter asserted therein (eg, the cause of the plaintiff’s condition).

An expert witness’s opinion can be based on a hearsay statement if other experts in the field would reasonably rely on that statement to form an opinion.

But the hearsay statement itself is inadmissible unless:

1) it is excluded or excepted from the rule against hearsay OR
2) its probative value in helping the jury evaluate the expert’s opinion substantially outweighs its prejudicial effect.

192
Q

Amount in Controversy

Singe Plaintiff vs. Single Defendant

A

If the diversity action involves only one plaintiff and one defendant, then the total value of all of the plainti!’s claims is calculated to determine the amount in controversy.

FRCP 18 allows a party to join (ie, add) as many claims as it has against an opposing party in a single lawsuit—even if the claims arise from different facts or occurrences*. But like all disputes in federal court, a suit involving multiple claims must satisfy subject-matter jurisdiction.

This can be established through either:

federal-question jurisdiction – when a claim arises under the U.S. Constitution, a treaty, or a federal law (not seen here) or

diversity jurisdiction – when the opposing parties are citizens of different states and the amount in controversy exceeds $75,000.

The amount in controversy refers to the amount demanded in the plaintiff’s complaint (excluding costs and interest). To meet the required amount, a single plaintiff may aggregate (ie, combine) joined claims asserted against a single defendant.

193
Q

Recovery under promissory estoppel is warranted if:

A

The equitable doctrine of promissory estoppel allows a party to recover damages stemming from another’s failure to perform a promise when there is no valid contract (as seen here).

Recovery under promissory estoppel is warranted if:

1) the promisor reasonably expected to induce reliance on the promise
2) the enforcing party reasonably relied on the promise
3) reliance on the promise caused the enforcing party to suffer substantial detriment AND
4) injustice can only be avoided by enforcing the promise.

194
Q

Can crossclaims only be asserted if they arise from the same transaction or occurence as another claim in the lawsuit?

A

YES.

A crossclaim is a claim for relief asserted against a coparty—eg, a defendant against a codefendant (as seen here).

Crossclaims are permissive (not compulsory) and can only be asserted if they arise from the same transaction or occurrence as another claim in the lawsuit.

195
Q

When will a law that interferes with the freedom of speech be struck down on its face?

A

The First Amendment free speech clause (applied to the states through the Fourteenth Amendment) protects the free flow of ideas with limited exceptions.

A law that infringes upon this right can be challenged on its face and struck down in its entirety under either:

1) the overbreadth doctrine – renders laws unconstitutional if they restrict a substantial amount of protected speech or
2) the vagueness doctrine – renders laws unconstitutional if ordinary persons would have to guess the law’s meaning because the law fails to provide notice of what speech is prohibited.

196
Q

Self-authenticating tangible evidence

A

Some tangible evidence is self-authenticating and can be admitted without extrinsic evidence of authenticity—

i) trade inscriptions affixed in the course of business that indicate ownership,
ii) official publications issued by a public authority (Public documents with official’s signature & certification by second official or seal)
iii) newspapers, periodicals & official publications
iv) Certified copies of public records & records of regularly conducted activities
v) Commercial paper with signature & related documents

197
Q

Good Samaritan Statute

A

A person who voluntarily aids or rescues another has a duty to act with reasonable ordinary care in the performance of that aid or rescue.

The state’s Good Samaritan statute exempts medical professionals from liability for ordinary negligence but not gross negligence.

Example: The facts here, including the doctor’s intoxication, would likely constitute gross negligence, and the state’s statute would not insulate the doctor from liability.

198
Q

Does a life sentence of a minor without the possibility of parole constitute cruel and unusual punishment for a non-homicide crime?

A

A sentence of life imprisonment without the possibility of parole constitutes “cruel and unusual punishment” under the Eighth Amendment when the sentence is imposed for a non-homicide crime on a defendant who was a minor at the time that the crime was committed.

199
Q

Can deadly force be used in the defense of property?

A

Generally, deadly force may not be used in the defense of property.

200
Q

Is citizenship status subject to strict scrutiny?

A

YES. The strict scrutiny standard typically applies if the governmental action involves a fundamental right or suspect classification, including citizenship status.

When reviewing government action under equal protection theories, a court applies one of three levels of review, depending on the classification of persons or the type of right concerned.

A court will generally apply the strict scrutiny test and strike down state laws that discriminate against resident noncitizens by requiring U.S. citizenship, such as laws prohibiting resident noncitizens from owning land, obtaining commercial fishing licenses, or being eligible for welfare benefits or civil service jobs.

201
Q

Can Fourth Amendment rights be asserted vicariously?

A

NO. Fourth Amendment rights are personal and may not be asserted vicariously.

A defendant cannot successfully challenge governmental conduct as a violation of the Fourth Amendment protection against unreasonable searches and seizures unless the defendant himself has been seized or he has a reasonable expectation of privacy with regard to the place searched or the item seized.

202
Q

Can a tenant waive the warranty of habitability?

A

NO, the warranty of habitability generally cannot be waived by the tenant, either by express language in the lease or by taking possession of the property with knowledge of the conditions.

203
Q

What is a tenant required to do when the landlord is in breach of the warranty of habitability?

A

The tenant is generally required to notify the landlord of conditions that constitute a breach of this warranty and give the landlord a reasonable opportunity to correct the problem.

For most residential leases, an implied warranty of habitability is imposed on the landlord.

The landlord’s failure to comply with applicable housing code requirements constitutes a breach of this warranty, particularly with regard to violations that substantially threaten the tenant’s health and safety.

204
Q

Are science experiments permitted in the courtroom as a demonstration?

A

A court may allow demonstrations and experiments to be performed in the courtroom.

Science experiments are permitted but may be excluded if they will result in an undue waste of time or confusion of the issues.

205
Q

What is robbery?

A

Robbery is larceny from the person or presence of the victim by force or intimidation.

Larceny is the:

(i) trespassory
(ii) taking and carrying away
(iii) of the personal property of another
(iv) with the intent to steal.

206
Q

Slander

A

A plaintiff may bring an action for slander, which typically is spoken rather than written defamation, if:

(i) the defendant’s defamatory language,
(ii) is of or concerning the plaintiff,
(iii) is published to a third party who understands its defamatory nature, and
(iv) it damages the plaintiff’s reputation.

For matters of public concern, the plaintiff is constitutionally required to prove fault on the part of the defendant.

If the plaintiff is either a public official or a public figure, then the plaintiff must prove actual malice.

207
Q

Does federal law preempt state law when the federal law intends to occupy the entire field?

A

YES

When Congress intends for a federal law to occupy the field, such as is the case with regard to the registration of aliens in the United States, federal law preempts state law dealing with the same subject even if federal law does not expressly preempt state law with regard to that subject.

NOTE: Implied preemption of state law based on federal occupation of the field reflects a congressional decision to foreclose any state regulation in the area, even if it is parallel to federal standards.

  • Even if federal law does not expressly preempt state law with regard to a certain subject, federal law can impliedly preempt state law when Congress intended for federal law to occupy the field.
208
Q

What is a “savings clause”?

A

Federal law may also contain “savings clauses” that explicitly preserve or allow state laws that regulate in the same area,

  • e.g., 33 U.S.C. § 1365 (The Clean Water Act preserves “any right which any person (or class of persons) may have under any statute or common law.”).
209
Q

Frustration of Purpose

A

The doctrine of frustration of purpose applies when an unexpected event arises that destroys one party’s purpose in entering into the contract, even if performance of the contract is not rendered impossible.

In order for a contractual obligation to be discharged by frustration of purpose, there must be:

i) an unexpected event that occurs, even though the nonoccurrence of the event was a basic assumption of the contract,
ii) without the fault of the frustrated party and
iii) the event destroys that party’s purpose in entering into the contract.

210
Q

Can a mortgagee elect to pursue its right against the mortgagor’s personal obligation without first seeking to foreclose on the mortgaged property?

A

YES. (in most states)

In most states, the mortgagee may elect to pursue its rights against the mortgagor’s personal obligation without first seeking to foreclose on the mortgaged property.

Alternatively, where the mortgagee has foreclosed on the mortgage and the mortgage obligation has not been completely satisfied, the mortgagee may seek the deficiency from the mortgagor herself.

211
Q

Can a defendant in a criminal case introduce evidence of his good character as being inconsistent with the type of crime charged?

A

YES. While generally character evidence is inadmissible to show that a person acted in conformity with a character trait, a defendant in a criminal case may introduce evidence of his good character as being inconsistent with the type of crime charged.

NOTE: While a party may not bolster the credibility of a witness until that witness has been attacked, a defendant may introduce evidence of his good character as being inconsistent with the type of crime charged.

212
Q

When must a court take judicial notice of an adjudicative fact?

A

Under FRE 201, the court must take judicial notice of an adjudicative fact if a party so requests and provides the court with the necessary information to do so.

213
Q

At what stages can judicial notice be taken?

A

Judicial notice may be taken at any stage of the proceeding, except against a criminal defendant for the first time on appeal.

214
Q

Does prosecution of a greater crime after the defendant has been sentenced with regard to a lesser included offfense violate the due process clause?

A

Generally prosecution of a greater crime, such as burglary, after the defendant has been sentenced with regard to a lesser included offense, such as criminal trespass, which is based on the same conduct as the greater offense, violates the Double Jeopardy Clause.

However, the Double Jeopardy Clause does not prohibit such prosecution where the defendant had been charged with the greater offense at the time that the defendant pleads guilty to the lesser included offense.

215
Q

When can consideration be found in a modification of a services contract?

A

Consideration can be found when there is a change in a party’s duties, even where that change is financially beneficial to the party.

216
Q

Who are the only parties that are subject to the “bulge provision” of FRCP 4?

A

The “bulge provision” in the federal rules that provides for service of process on a party within 100 miles of the forum court even if state law would not otherwise permit such service only applies to a third-party defendant joined under Rule 14 or a required party joined under Rule 19.

217
Q

Can a defendant in a criminal case who testifies on his own behalf be impeached by a juvenile adjudication to shat that the defendant is untruthful?

A

NO.

The defendant in a criminal case who testifies on his own behalf may not be impeached by a juvenile adjudication to show that the defendant is untruthful.

**although in limited circumstances a non-defendant witness in a criminal case may be impeached with a juvenile adjudication for an offense, this exception does not apply to a defendant in a criminal case.**

  • A criminal defendant may impeach a witness by introducing evidence of a witness’s juvenile adjudication to show motive to lie or bias.
  • (Note: A criminal defendant may also impeach a witness’s character for truthfulness with the witness’s juvenile adjudication when an adult’s conviction for the offense would be admissible to attack the adult’s credibility and admitting the evidence is necessary to fairly determine guilt or innocence.)
218
Q

Can a federal court assert supplemental jurisdiction over a counterclaim that does not satisfy the jurisdictional amount when the counterclaim is compulsory?

What about a permissive counterclaim?

A

A federal court may assert supplemental jurisdiction over a counterclaim that does not satisfy the jurisdictional amount when the counterclaim is compulsory.

However, a permissive counterclaim does not qualify for supplemental jurisdiction and therefore must satisfy the jurisdictional amount and the rule of complete diversity.

219
Q

An implied reciprocal servitude must meet the following requirements to be enforceable:

A

An implied reciprocal servitude must meet the following requirements to be enforceable:

(i) there must be intent to create a servitude on all real property interests (i.e., a common scheme);
(ii) the servitude must be negative (i.e., a promise to refrain from doing something); and
(iii) the party against whom enforcement of the servitude is sought must have actual, record, or inquiry notice.

Reciprocal negative servitudes are implied from the common scheme.

220
Q

Does the prosecution have a constitutional duty to disclose to the defense any material evidence favorable to the defendant and relevant to the prosecution’s case in chief that would negate guilt or diminish culpability or punishment?

A

YES.

The prosecution has an affirmative duty to disclose to the defendant any material evidence favorable to the defendant and relevant to the prosecution’s case in chief that would negate guilt or diminish culpability or punishment.

Failure to disclose such evidence (known as Brady material) violates the Due Process Clause of the Fourteenth Amendment.

221
Q

Is intoxication a valid defense to a general intent crime such as manslaughter?

A

NO.

Intoxication is not a valid defense to a general intent crime such as manslaughter.

However, there must be a causal connection between the unlawful act and the death for involuntary manslaughter to apply.

***Voluntary intoxication is a defense to specific-intent crimes if the intoxication prevents the formation of the required intent.

  • Involuntary manslaughter is an unintentional homicide committed with criminal negligence or during an unlawful act, and does not require any specific intent.
222
Q

What is an exception to defending one’s property by use of deadly force?

A

Although generally there is no right to use deadly force in defense of property, a person may use deadly force:

i) to prevent or terminate forcible entry into a dwelling if the occupant reasonably believes that the intruder intends to commit a felony inside.

However, the use of deadly force against an intruder exiting the dwelling is generally not permissible.

223
Q

Can jail administrators require all arrestees committed to the general population of a jail to undergo a no-touch visual strip search?

A

Yes. Jail administrators may require all arrestees committed to the general population of a jail to undergo a no-touch visual strip search, even if the arrest was for a minor offense and even in the absence of reasonable suspicion that the arrestee possesses a concealed weapon or other contraband.

224
Q

Article III, Section 2 delineates the jurisdiction of federal courts as limited to cases or controversies:

A

i) Arising under the Constitution, laws, and treaties of the United States;
ii) Affecting foreign countries’ ambassadors, public ministers, and consuls;
iii) Involving admiralty and maritime jurisdiction;
iv) When the United States is a party;
v) Between two or more states, or between a state and citizens of another state;
vi) Between citizens of different states or between citizens of the same state claiming lands under grants of different states; or
vii) Between a state, or its citizens, and foreign states, citizens, or subjects.

225
Q

When is a TRO appealable?

A

In general, a TRO is not an appealable interlocutory order unless it is extended beyond the period permit under Rule 65.

Under Rule 65, a court is generally permitted to grant a TRO for a period of no more than 14 days in the absence of the existence of good cause or the consent of the adversary.

A TRO that is extended beyond this time limit will be treated as a preliminary injunction and therefore be appealable.

226
Q

Can a court grant a TRO ex parte?

A

YES

A court may grant a TRO on an ex parte basis when the moving party can establish, under written oath, that immediate and irreparable injury will result prior to hearing the adverse party’s opposition and the movant’s attorney certifies in writing any efforts made to give notice and the reason why notice should not be required.

227
Q

If a criterion for the issuance of a TRO was falsely established, is the TRO immediately appealable?

A

NO.

Although a criterion for the issuance of a TRO was falsely established, the granting of TRO is not immediately appealable because such an order is an interlocutory order.

228
Q

What takes precedence when a federal statute conflicts with an executive agreement?

What about a federal statute vs. a federal treaty?

A

When a federal statute conflicts with an executive agreement, the federal statute takes precedence over the executive agreement.

A federal statute and a federal treaty are roughly equivalent. When the two conflict, whichever was enacted more recently would govern.

229
Q

Firefighter’s Rule

A

Often known as the “firefighter’s rule,” an emergency professional, such as a police officer or firefighter, is barred from recovering damages from the party whose negligence caused the injury if the injury results from a risk inherent in the job.

230
Q

Deadly force is justified in self-defense when the defendant…

A

Deadly force is justified in self-defense when the defendant reasonably believes danger of death or serious bodily harm is immediate.

NOTE: a claim of self-defense requires only a reasonable belief of imminent harm. It does not require proof that the attacker had the actual intention to kill the defendant.

231
Q

How is diversity of citizenship tested with respect to limited partnerships?

A

In testing whether this requirement is satisfied with respect to a limited partnership, the citizenship of each member of the partnership, limited partners as well general partners, must be taken into account.

232
Q

Burglary is…

A

Burglary is the

(i) breaking and
(ii) entering
(iii) of the dwelling
(iv) of another
(v) at nighttime
(vi) with the specific intent to commit a felony therein.

NOTE: Because the crime of burglary is complete when a defendant breaks and enters a dwelling at night with the intent to commit a felony therein, a defendant’s subsequent commission of larceny with regard to a different item of personal property is irrelevant with regard to the burglary charge.

233
Q

What is voluntary manslaughter?

A

Voluntary manslaughter is murder committed in response to adequate provocation; that is, the defendant was provoked by a situation that could inflame the passion of a reasonable person to the extent that it could cause that person to momentarily act out of passion rather than reason.

Generally, the discovery of a spouse’s adultery constitutes adequate provocation.

234
Q

Can law enforcement use a device that is not available to the general public and that is capable of detecting lawful activity?

A

NO.

While not necessarily a winning argument, the Supreme Court held in Kyllo v. US that the use of a device that is not available to the general public (a heat-sensing device) and that is capable of detecting lawful activity can constitute an unlawful search.

Example:

Here, the device, a passive alcohol sensor, is not generally available to the public and is capable of detecting lawful activity; an adult driver is not prohibited from having consumed any alcohol prior to driving but instead may be punished only for driving while under the influence of alcohol.

In addition, the court noted that the heat-sensing device gave the officer information that he could not have gained without entering the house.

Here, the passive alcohol sensor gave the officer information about the presence of alcohol that he, through the use of his own senses, would not have had.

235
Q

Does recording a deed raise the rebuttable presumption of delivery?

A

YES.

Recording a deed raises the rebuttable presumption of delivery. Acceptance is treated as occurring at the time of the transfer.

236
Q

Does the execution of a deed after a will give it priority over the will?

A

NO.

The fact that a deed was executed after a will does not give it priority over the will. Mere execution of a deed is not sufficient to transfer title; delivery and acceptance are also required.

237
Q

What is a suretyship?

A

Suretyship is a three-party contract, wherein one party (the surety) promises a second party (the obligee) that the surety will be responsible for any debt or other obligation of a third party (the principal) resulting from the principal’s failure to pay as agreed.

Under the Statute of Frauds, a suretyship contract, or contract to answer for the debt of another, must be signed by the party to be charged.

238
Q

What does the doctrine of issue preclusion do?

A

The doctrine of issue preclusion, often called “collateral estoppel,” precludes the re-litigation of issues of fact or law that have already been necessarily determined by a judge or jury in an earlier action.

For this doctrine to apply:

1) the issue sought to be precluded must be the same as that involved in the prior action,
2) must have been actually litigated in the prior action, and
3) must have been essential to the prior valid, final judgment.

239
Q

Can a bona fide purchaser of a servient estate make an easement unenforceable if they do NOT have notice of it?

A

YES.

A sale of the servient estate to a bona fide purchaser (i.e., a purchaser without notice) of the easement can make the easement unenforceable.

NOTE: Even if the easement is not in the deed but it is properly recorded, this provides constructive notice to the BFP and it will be enforceable against the BFP.

240
Q

Are changes in easement use permissible?

A

Changes in use are examined for reasonableness, as there is the assumption that the original parties contemplated the easement’s present and future use.

241
Q

Is a member of a class action who has the option of opting out of the settlement but fails to do so bound by the settlement agreement approved by the court?

A

YES.

When a member of a class who has the option of opting out of the settlement fails to do so, that member is bound by a settlement agreement approved by the court.

Even though a class member has objected to a proposed settlement that is subsequently approved by the court, that class member can only avoid the consequences of the settlement by opting out.

Mere objection to the settlement is not sufficient to escape its binding effect.

242
Q

Can a party recover for work performed even in the event of that party’s breach?

A

YES.

A party can generally recover for work performed, even in the event of that party’s breach.

Example:

Here, the website would be unjustly enriched if the production company is unable to recover the reasonable value of its services.

Even though the express condition (high-definition videos) failed and thereby relieved the website of its duty to perform under the contract, the production company can recover in restitution for the reasonable value of the standard-definition videos because the website used them and thus would be unjustly enriched by retaining this benefit.

243
Q

To recover under a strict products liability theory of recovery, the plaintiff must prove that….

A

To recover under a strict products liability theory of recovery, the plaintiff must prove that the product was defective, the defect existed at the time the product left the defendant’s control, and the defect caused the plaintiff’s injuries when the product was used in an intended or reasonably foreseeable way.

244
Q

Can a defendant be convicted of felony murder even if the killing was accidental?

A

YES.

It does not matter that the death was accidental. Felony murder is an unintended killing proximately caused by and during the commission or attempted commission of an inherently dangerous felony. Robbery is one such inherently dangerous felony.

Example:

The underlying felony merges into the crime of felony murder, so here, the robbery merges into the felony murder and the man can only be convicted of and punished for felony murder, not both felony murder and robbery.

As an accessory before the fact, the girlfriend is liable to the same extent as the man.

245
Q

A defendant is liable for intentional infliction of emotional distress (IIED) if he:

A

intentionally or recklessly acts with extreme and outrageous conduct that causes the plaintiff severe emotional distress.

246
Q

The Fifth Amendment Double Jeopardy Clause provides protection against:

A

The Fifth Amendment Double Jeopardy Clause provides protection against:

(i) a second prosecution for the same offense after acquittal;
(ii) a second prosecution for the same offense after conviction; and
(iii) multiple punishments for the same offense.

However, when jeopardy has attached with respect to a lesser included offense prior to the occurrence of an event necessary to establish the greater offense, the defendant may be subsequently tried for the greater offense.

247
Q

What level of scrutiny applies to state laws that restrict or prohibit a resident alien’s participation in government functions?

A

Generally, the strict scrutiny test applies to state laws that discriminate against resident aliens.

However, a growing exception exists requiring only a rational relationship to a legitimate state interest for state laws that restrict or prohibit a resident alien’s participation in government functions.

248
Q

Can the state conduct warrantless searches of businesses in highly regulated industries due to an urgent public interest?

A

YES.

The state may conduct warrantless searches of businesses in highly regulated industries due to an urgent public interest. This is due to the theory that the business impliedly consented to warrantless searches by entering into a highly regulated industry.

249
Q

Evidence Obtained From Administrative Searches

What are the two kinds of administraative searchs where Police do not need search warrants to conduct?

A

Two kinds:

1) Administrative warrants (e.g., fire or health inspections of a building)
2) Warrantless administrative searches—used to ensure compliance with various administrative regulations; examples:
i) Airplane boarding areas
ii) International borders
iii) Highly regulated industries (liquor stores, gun shops, etc.)
iv) Searches of students in public schools
v) Special needs searches; e.g., drug testing of railroad employees after an accident
vi) Roadblocks for drunk driving or seeking information

250
Q

Is a mortgage protecting by recording acts if it is not given simultaneously with the loan?

A

NO.

Mortgagees are considered to have “paid value” and are protected by the recording acts, unless the mortgage is not given simultaneously with a loan (such as when the mortgage is given after the loan is created).

Unless the recording act governs the priority of real property interests, the common law rule of “first in time, first in right” applies.

Example:

Here, the applicable recording act is a race type recording act.

Although the uncle did record his mortgage first, he is not considered to have “paid value” for his mortgage because he had made the loan to the buyer several months before he obtained the mortgage.

Consequently, because the uncle is not a person who is protected by the recording act, the bank’s mortgage has priority over the uncle’s mortgage under the common law “first in time, first in right” rule.

251
Q

What is the role of the jury if there are both legal and equitable claims?

A

The right to trial by jury is evaluated for each claim.

If an action involves legal and equitable claims that raise issues of fact common to both, then under the Seventh Amendment, the jury must be permitted to determine these issues by trying the legal claim first.

Thereafter, the court determines the equitable claim, but it is bound by the jury’s findings of fact on the legal claim.

252
Q

The standard of review to which governmental action is subjected when challenged on substantive due process grounds depends on the right asserted by the plaintiff. If the plaintiff is asserting a fundamental right, the government must establish that its requirement is the least restrictive means to achieve a compelling governmental interest.

If a fundamental right is not involved, the plaintiff must show that the restriction…

A

If a fundamental right is not involved, the plaintiff must show that the restriction does not bear a rational relationship to a legitimate government interest.

Laws are presumed valid under this standard, so the burden is on the challenger to overcome this presumption by establishing that the law is arbitrary or irrational.

253
Q

Is a hotel clerk’s consent to a governmental search of a room during the time it is rented sufficient to justify the search?

A

NO.

As with the search of a home, the search of a motel room by a government agent may be an unreasonable search.

A hotel clerk’s consent to a governmental search of a room during the time it is rented is insufficient to justify the search.

Accordingly, in the absence of an exception to the warrant requirement, the police needed a warrant to search the hotel room.

254
Q

Is character evidence generally admissible or inadmissible?

A

While character evidence (i.e., generalized information about a person’s behavior) can be admissible for impeachment purposes, such as to show the witness’s character of untruthfulness, character evidence is generally inadmissible.

255
Q

Employers are not liable for the torts of true independent contractors, except where:

A

Employers are not liable for the torts of true independent contractors, except where:

(i) an employer is negligent in selecting an independent contractor,
(ii) the function is inherently dangerous,
(iii) the employer is a storekeeper or operator of premises open to the public and has the duty to keep the premises in a reasonably safe condition, or
(iv) the function is non-delegable.

256
Q

Are hearings on preliminary matters, such as the admissibility of a confession, required to be conducted outside the presence of the jury?

A

YES.

The trial judge generally decides preliminary questions regarding the competency of evidence, including the admissibility of evidence, whether a privilege exists, and whether a person is qualified to be a witness.

Hearings on preliminary matters must be conducted outside the presence of the jury:

i) when the hearing involves the admissibility of confessions,

ii) when a defendant in a criminal case is a witness and so requests, or
iii) when justice requires it.

257
Q

Non-delegation doctrine

A

Because Congress is vested by Article I with “all legislative powers,” it may not delegate that power to any other branch of government. This principle is known as the “nondelegation doctrine.”

However, delegation of some of Congress’s authority to the executive branch has consistently been held constitutional, so long as Congress specifies an “intelligible principle” to guide the delegate.

258
Q

Doctrine of Transferred Provocation

A

Under the doctrine of transferred provocation, when a defendant accidentally kills the wrong person, he will be guilty of voluntary manslaughter if that would have been his crime had he killed the provoker.

Voluntary manslaughter is murder committed in response to adequate provocation (i.e., in the “heat of passion”); that is, the defendant was provoked by a situation that could inflame the passion of a reasonable person to the extent that it could cause that person to momentarily act out of passion rather than reason.

  • Generally, a serious battery constitutes adequate provocation.
259
Q

What must a government entity show in order for a governmental affirmative action program based on race to survive?

A

For a governmental affirmative action program based on race to survive, the relevant governmental entity (here, Congress) must show more than a history of societal discrimination.

The government must itself be guilty of specific past discrimination against the group it is seeking to favor, and the remedy must be narrowly tailored to end that discrimination and eliminate its effects. (strict scrutiny)

In other words, the elimination of past discrimination in a particular governmental institution is a compelling state interest; attempting to remedy general societal injustice through affirmative action is not.

260
Q

Do police need a warrant to search a car if they have probable cause to believe it contains contraband?

A

NO. The Fourth Amendment does not require police to obtain a warrant to search a vehicle if they have probable cause to believe that it contains contraband or evidence of a criminal activity.

So long as they have probable cause to do so, the police may search anywhere in a car that they believe there to be contraband, including the trunk and locked containers that could reasonably hold the evidence sought.

261
Q

Does paying a discounted price waive the implied warranty of merchantability?

A

NO. Paying a discounted price does not waive the implied warranty of merchantability.

262
Q

Would purposefully serving someone poison qualify as acting with a depraved heart, even if the wife truly did not meant to cause her husband’s death?

A

YES. Purposefully serving someone poison would certainly qualify as acting with a depraved heart, even if the wife truly did not meant to cause her husband’s death.

263
Q

Specific personal jurisdiction

A

When a cause of action arises out of or closely relates to a defendant’s contact with the forum state, jurisdiction may be warranted over that action even if that contact is the defendant’s only contact with the forum state.

This type of jurisdiction is referred to as “specific personal jurisdiction.”

Due process concerns are met if the exercise of jurisdiction is reasonably foreseeable by the foreign defendant.

Example from this question:

The fact that the negotiations for the management agreement occurred in the foreign country is not dispositive as to personal jurisdiction.

Even though there might be jurisdiction over the case in the foreign country, that does not determine whether there are minimum contacts supporting personal jurisdiction in the forum state.

264
Q

Doctrine of joint and severally liability

A

Under the doctrine of joint and several liability, each of two or more tortfeasors who is found liable for a single and indivisible harm to the plaintiff is subject to liability to the plaintiff for the entire harm.

Thus, a plaintiff can collect the full damages to which he is entitled from any of the defendants.

Under a system of pure comparative negligence, a plaintiff’s damages are reduced by the proportion that his fault bears to the total harm.

265
Q

Common-law murder is the:

A

Common-law murder is the:

(i) unlawful (i.e., without a legal excuse)
(ii) killing
(iii) of another human being
(iv) committed with malice aforethought.

“Malice aforethought” includes the intent to kill, the intent to inflict serious bodily injury, reckless indifference to an unjustifiably high risk to human life (i.e., depraved-heart murder), or the intent to commit certain felonies (i.e., felony murder).

In most jurisdictions, the requisite depravity for depraved-heart murder does not exist when a defendant is actually unaware of the risk involved in the conduct.

Example: Here, the homeowner is guilty of depraved-heart murder because he set the fire even though he was subjectively aware of the great risk that someone could die as result of the burning of his residence, and the firefighter died as a result.

266
Q

How does the Blockburger test apply for double jeopardy purposes?

A

Under the Fifth Amendment Double Jeopardy Clause that is applicable to the states through the Fourteenth Amendment, a person may not be prosecuted for the same offense after being acquitted of that offense.

The Blockburger test is applied to determine whether the crimes constitute the same offense for double jeopardy purposes.

Under this test, each crime must require the proof of an element that the other does not in order for each to be considered as a separate offense.

A lesser-included offense is one that does not require proof of an element beyond those required by the greater offense.

For example, false imprisonment is a lesser-included offense of kidnapping. Accordingly, a defendant cannot be prosecuted for kidnapping if he was previously acquitted of the lesser offense of false imprisonment.

267
Q

Procedural due process only applies in what settings?

A

Procedural due process only applies in quasi-judicial or adjudicatory settings, and NOT with respect to the adoption of general legislation.

Example:

Here, the professor was not terminated as an individual, but instead as a consequence of the abolition by the legislature of the specific university at which he was granted tenure.

Therefore, his termination did not trigger his procedural due process rights.

268
Q

Will a court modify a contract for unconscionability that was fair when entered if unforeseeable intervening events make the contract seem unconscionable?

A

NO.

A court may modify or refuse to enforce a contract or part of a contract on the ground that it is unconscionable, that is, when it is so unfair to one party that no reasonable person in the position of the parties would have agreed to it.

The contract or part of the contract at issue must have been offensive at the time it was made.

NOTE: foreseeability is not a relevant factor in an unconscionability inquiry.

Example:

The fact that intervening events caused prices to rise only means that the farmer did not receive as good a deal as he could have if he had waited; this, however, is the risk inherent in business dealings and does not make the transaction unconscionable.

269
Q

Can a 17 year old defendant be subject to the death penalty?

A

NO.

The Eighth Amendment prohibition against “cruel and unusual punishment” prohibits the execution of a defendant who was younger than eighteen years of age at the time of the commission of a crime.

270
Q

Does law enforcement violate the knock and announce rule if they fail to knock and announce before executing a valid arrest warrant?

A

YES.

Even with a warrant, most states and federal law mandate that a police officer who is making an arrest in a home must generally knock and announce his purpose before entering the home.

while having a reasonable belief that the individual is home is a requirement to entering an individual’s home to arrest him, that is not enough, and the officer would still need to knock and announce.

271
Q

What type of oral suretyship contracts can be enforeced?

A

The Statute of Frauds applies to suretyship agreements (i.e., one person’s promise to pay the debts or be responsible for an obligation of another).

Note that some oral suretyship contracts can be enforced:

  • indemnity contracts and contracts wherein the surety’s main reason for paying the debt is the surety’s own economic advantage.
272
Q

When does a landlord breach the covenant of quiet enjoyment?

A

Every lease (both commercial and residential) contains an implied covenant of quiet enjoyment.

This covenant is a promise by the landlord not to interfere with the tenant’s possession of the leased premises.

It is breached only when the conduct of the landlord or someone with superior title prevents the tenant from possessing the leased premises.

The landlord is not liable for acts of other tenants, but he has a duty to take action against a tenant’s nuisance-like behavior and to control the common areas. Off premises actions of third parties will not suffice.

Any actions by the landlord that breach this covenant may amount to an actual or constructive eviction of the tenant.

If the landlord removes the tenant from the premises, then the total eviction terminates the lease and ends the tenant’s obligation to pay rent.

1). Partial eviction

If the tenant is prevented from possessing or using a portion of the leased premises, then the tenant may seek relief for a partial actual eviction. The type of relief granted depends on who prevented the possession.

a) Landlord

The tenant is completely excused from paying rent for the entire premises if the landlord is responsible for partial eviction.

b) Third parties

The tenant must pay the reasonable rental value of the premises occupied if the partial eviction is by a third party with a superior claim to the property.

The tenant is not excused from paying rent if a third-party adverse possessor/trespasser partially evicts the tenant.

Constructive Eviction

If the landlord breaches a duty to the tenant, such as failing to make a repair, that substantially interferes with the tenant’s use and enjoyment of the leasehold (e.g., fails to provide heat or water), then the tenant’s obligation to pay rent is excused due to constructive eviction only if the tenant gives notice and adequate time to permit the landlord to fulfill his duty and vacates the property within a reasonable amount of time.

Not every interference with the use and enjoyment of the premises amounts to a constructive eviction. Temporary or de minimis acts not intended to amount to a permanent expulsion do not amount to constructive eviction.

273
Q

Does habit evidence need to be sufficiently specific?

A

YES.

In order for evidence of a person’s conduct to be admissible as habit evidence, the evidence must be sufficiently specific. A tendency to cross the street carelessly is not likely a sufficiently regular response to a particular situation to qualify as a habit

274
Q

Contract rights are assignable unless…

A

Generally, contract rights are assignable unless the assignment materially increases the duty or risk of the obligor or materially reduces the obligor’s chance of obtaining performance.

Example:

Here, because the right assigned is the right to receive payment and the songwriter has performed his obligation under the contract by composing the song, this right may be assigned and enforced by the charity through a breach of contract action.

275
Q

Under the dormant commerce clause, when does a state or local regulation discriminate against out-of-state commerce?

A

A state or local regulation discriminates against out-of-state commerce if it protects local economic interests at the expense of out-of-state competitors.

NOTE:

The mere fact that the entire burden of a state’s regulation falls on an out-of-state business is not sufficient to constitute discrimination against interstate commerce.

The Dormant Commerce Clause protects the interstate market, not particular interstate firms, from prohibitive or burdensome regulations.

A state regulation that is not discriminatory may still be struck down as unconstitutional if it imposes an undue burden on interstate commerce.

276
Q

Can a state behave in a discriminatory fashion if it is acting as a market participant?

A

Yes. A state may behave in a discriminatory fashion if it is acting as a market participant (buyer or seller), as opposed to a market regulator.

If the state is a market participant, it may favor local commerce or discriminate against nonresident commerce as could any private business.

Example:

Here, Section 3 requires that State A buy goods only from environmentally friendly vendors located within State A. Because State A is buying goods, it is a market participant and may discriminate against nonresident commerce as could any private business. Thus, Section 3 does not violate the Constitution.

277
Q

The Dormant Commerce Clause Rule Statement

A

The Dormant Commerce Clause is a doctrine that limits the power of states to legislate in ways that impact interstate commerce.

If Congress has not enacted legislation in a particular area of interstate commerce, then the states are free to regulate, so long as the state or local action does not:

(i) discriminate against out-of-state commerce,
(ii) unduly burden interstate commerce, or
(iii) regulate extraterritorial (wholly out-of-state) activity.

278
Q

What does the Eleventh Amendment do?

A

The Eleventh Amendment is a jurisdictional bar that prohibits the citizens of one state from suing another state in federal court.

It immunizes the state from suits in federal court for money damages or equitable relief when the state is a defendant in an action brought by a citizen of another state.

EXCEPTIONS:

There are a few notable exceptions, including when a state waives its immunity under the Eleventh Amendment.

279
Q

Superseding cause

A

A superseding cause is any intervening cause that breaks the chain of proximate causation between the defendant’s tortious act and the plaintiff’s harm, thereby preventing the original defendant from being liable to the plaintiff.

Most courts hold that an unforeseeable intervening cause is a superseding cause that breaks the chain of causation between the defendant and the plaintiff.

280
Q

The doctrine of vicarious liability

A

The doctrine of vicarious liability asserts that a principal is liable for the acts of an agent, even though the principal is innocent of fault and not directly guilty of any tort or crime.

281
Q

The Doctrine of Respondeat Superior

A

Under the doctrine of respondeat superior, a principal may be vicariously liable for a tort committed by an agent acting within the scope of his employment. The principal is liable despite the absence of tortious conduct by the principal. This is also known as derivative liability.

An employee acts within the scope of employment when either performing work assigned by the employer or engaging in a course of conduct subject to the employer’s control. When an employee acts independently of any intent to serve any purpose of the employer, the employer may escape liability.

282
Q

Proximate Cause

A

The majority rule for proximate cause requires the plaintiff suffer a foreseeable harm that is not too remote and is within the risk created by the defendant’s conduct.

An intervening cause is a factual cause of the plaintiff’s harm that contributes to her harm after the defendant’s tortious act is completed.

283
Q

A nonparty has the right to intervene in an action when, upon timely motion….

A

Under FRCP 24, a nonparty has the right to intervene in an action when, upon timely motion,

(1) the nonparty has an interest in the subject matter of the action,
(2) the disposition of the action may impair the nonparty’s interests, and
(3) the nonparty’s interest is not adequately represented by existing parties.

The burden is on the party seeking to intervene.

284
Q

What is the effect of a TRO?

A

A TRO preserves the status quo of the parties until there is an opportunity to hold a full hearing on whether to grant a preliminary injunction.

A TRO has immediate effect and lasts no longer than 14 days unless good cause exists.

A TRO can be issued without notice to the adverse party if the moving party can show

(1) that immediate and irreparable injury will result prior to hearing the adverse party’s arguments and
(2) the efforts made at giving notice and the reason why notice should not be required.

Additionally, the party seeking aa TRO must post a bond to cover the costs in the event the TRO is issued wrongfully.

285
Q

Must a party seeking a TRO post a bond?

A

YES.

The party seeking aa TRO must post a bond to cover the costs in the event the TRO is issued wrongfully.

286
Q

A party seeking a preliminary injunction must establish that:

A

A preliminary injunction can be issued if the opponent is given notice and the court holds a hearing on the issue.

A party seeking a preliminary injunction must establish that:

(1) the party is likely to succeed on the merits;
(2) the party is likely to suffer irreparable harm in the absence of relief;
(3) the balance of equities is in his favor; and
(4) the injunction is in the best interest of the public.

Additionally, the party seeking the preliminary injunction must provide a bond to cover the costs in the event the preliminary injunction is issued wrongfully.

287
Q

Confidential martial communications privilege

vs.

Spousal Immunity

A

The confidential marital communications privilege covers confidential marital communications between spouses while they were married.

The privilege extends beyond the termination of the marriage. The majority view is that both spouses hold a marital communication privilege, and some have taken the position that the communicating spouse alone can assert the privilege.

  • Under the spousal immunity privilege, a married person may not be compelled to testify against his spouse in any criminal proceeding.*
  • The spousal immunity privilege can be asserted only during a valid marriage, and expires upon divorce.*
288
Q

SMJ / Diversity jurisdiction rule statement

A

Subject matter jurisdiction is a court’s competence to hear and determine cases of the general class and subject to which the proceedings in question belong. For a federal district court to have SMJ over a complaint, it must have either diversity jurisdiction or federal question jurisdiction.

For diversity jurisdiction to exist, both the amount-in-controversy and diversity-of-citizenship requirements must be met.

The AIC requirement mandates that the AIC must exceed $75,000. In general, a plaintiff’s good faith assertion in the complaint that the action satisfies the AIC is sufficient, unless it appears to be a legal certainty that the plaintiff cannot recover the amount alleged.

For the diversity of citizenship requirement to be met, there must be complete diversity between opposing parties when the action is filed. There is no diversity of citizenship if any plaintiff is a citizen of the same state as any defendant.

For an individual to be a citizen of a state, the individual must be a citizen of the US and a domiciliary of the state. An individual is a domiciliary of the state in which she is present and intends to reside for an indefinite period. An individual can only have one domicile at a time.

The presumption is that a place of domicile continues until it is definitively changed. An intent to move without relocating to another state will not result in a change in domicile.

289
Q

Personal Jurisdiction Rule Statement

A

In addition to having SMJ, a court must be able to exercise judicial power over the persons or property involved in the cases or controversies before it, this is known as personal jurisdiction. A federal district court must generally determine PJ as if it were a court of the state in which it is situated. Thus, a federal court will look to state jurisdictional statutes to determine if it has personal jurisdiction.

The Due Process Clause of the Fourteenth Amendment permits a court to exercise personal jurisdiction over a defendant if the defendant has “minimum contacts” with the forum state such that the maintenance of the action does not offend traditional notions of fair play and substantial justice.

A defendant’s contacts with the forum state must be purposeful and substantial, such that the defendant should reasonably foresee being taken to court there. Foreseeability depends on whether a defendant recognizes or anticipates that by running his business, he risks being party to a suit in a particular state.

Specific Jurisdiction

When a cause of action arises out of or closely relates to a defendant’s contact with the forum state, jurisdiction may be warranted over that action even if that contact is the defendant’s only contact with the forum state. This type of jurisdiction is often referred to as “specific personal jurisdiction.”

For specific personal jurisdiction to exist, there must be an affiliation between the forum and the underlying controversy, activity or an occurrence that takes place in the forum State and is therefore subject to the State’s regulation.

General Jurisdiction

To determine whether the corporation is subject to general personal jurisdiction, the proper inquiry is whether a corporation’s affiliations with the forum state are so “continuous and systematic” as to render the corporation essentially “at home” in the forum state.

A corporate defendant is always at home in the state of the corporation’s place of incorporation and the state of its principal place of business. In exceptional cases, a corporate defendant’s operations in another forum may be so substantial and of such a nature as to render the corporation also at home in that state.

290
Q

Can one co-tenant bind another co-tenant to a boundary line agreement?

A

NO.

Unless there is an agreement to the contrary, one co-tenant may not bind another co-tenant to a boundary line agreement.

291
Q

Can a state require a local party to participate in an open primary?

A

NO. A state cannot require a local party to participate in an open primary (i.e., a primary in which any voter in the state may vote in a political party’s primary) to choose presidential electors where the national party has required that electors must be chosen only by party members.

Additionally, a state may not prohibit a political party from allowing independents to vote in its primary. States cannot prohibit political parties from allowing independents to vote in primaries.

Open primaries cannot be required by the state.

292
Q

When may a court grant a renewed motion for judgment as a matter of law?

A

A court may grant a renewed motion for judgment as a matter of law if the court finds that there is insufficient evidence for a jury reasonably to find for that party.

If the court finds that there is sufficient evidence for the jury to have reasonably found for a party, the court should not grant the motion.

293
Q

Is there a regulatory taking when a governmental regulation adversely affects a person’s property interest?

A

NO. Generally, a governmental regulation that adversely affects a person’s property interest is not a taking.

In the context of a regulation, a state or local government can act under its police power for the purposes of health, safety, welfare, aesthetic, and environmental concerns.

A taking occurs when the government takes title to land, physically invades land, or severely restricts the use of land.

294
Q

May a defendant amend its answer after the close of discovery to add a well-established affirmative defense?

A

Generally, the defendant’s answer must state any avoidance or affirmative defense that the defendant has, or that is deemed waived.

However, the rules also provide that pleadings can and should be amended by leave of the court when justice requires it.

Courts will generally permit the amendment unless it would result in undue prejudice to the opposing party.

295
Q

Motion for summary judgment rule statement

A

A motion for summary judgment should be granted if the pleadings, discovery, and disclosure materials on file, and any affidavits show that there is no genuine issue as to material fact and that the movant is entitled to judgment as a matter of law.

A genuine issue of material fact exists when a reasonable jury could return a verdict in favor of the non-moving party. In ruling on a motion for summary judgment, the court is to construe all evidence in the light most favorable to the non-moving party and resolve all doubts in favor of the non-moving party.

296
Q

Choice-of-law / noncompete clause rule statement

A

A federal court sitting in diversity must apply the substantive law of the state in which it sits, including the state’s choice of law rules.

If there is an express choice-of-law provision in the contract, then that law will generally govern unless there is no significant basis for the parties’ choice or it is contrary to public policy.

297
Q

Principal / Agent liability Rule statements

A

A principal is subject to liability on a contract if the agent acted with actual or apparent authority to bind the principal. Express actual authority can be created by oral or written words, clear direct and definite language, or specific detailed terms and instructions. For express actual authority to exist, the principal’s manifestation must cause the agent to believe that the agent is doing what the principal wants, and the agent’s belief must be reasonable.

Apparent authority derives from the reasonable belief of a third party on the agent’s authority based on the principal’s manifestation of that authority. (note: a third party’s reliance on an agent’s statement that she has actual authority to enter into the contract is insufficient to support apparent authority. The belief must come from the principal’s manifestation to the third party).

An agent is not a party to a contract if the agent discloses the existence and identity of the principal. However, an agent purporting to be acting for a principal gives an implied warranty of authority to the third party. If the agent lacks the power to bind the principal, then a breach of the implied warranty has occurred, and the agent is liable to the third party. (note: an agent is can be considered to not be a party to a contract but still breach the implied warranty of authority.)

A principal is undisclosed if the third party has no notice of the principal’s existence. An agent who enters into a contract on behalf of an undisclosed principal becomes a party to the contract. If the agent had the authority to bind the principal to the contract, then both the principal and the agent are parties to the contract with the third person.

Once the third party discovers the existence of the principal, the election of remedies doctrine requires the third party to choose to hold liable either the principal or the agent. The third party can only recover against one of them.

A principal is partially disclosed if the third party has notice of the principal’s existence but not the principal’s identity. Unless the agent and the third party agree otherwise, an agent who enters into a contract on behalf of a partially disclosed principal becomes a party to the contract.

If the agent binds the principal to the contract, or if the principal ratifies the contract, then both the principal and agent are parties to the contract with the third party.

If the principal ratifies the agent’s action, then the principal is bound just as if the action had been authorized, even if the agent acted without authority. Ratification occurs when a principal affirms a prior act that was done on the principal’s behalf.

The principal must (i) ratify the entire contract, (ii) have legal capacity to ratify the transaction, (iii) ratify in a timely manner, and (iv) have knowledge of the material facts involved in the original act.

298
Q

Brief Rule Statement on Agent’s express/implied actual authority / apparent authority

A

An agent has the power to bind the principal to a contract when the agent has actual or apparent authority.

Express actual authority can be created via written or spoken words, through clear language, or by specific detailed terms and instructions.

Implied actual authority allows an agent to take whatever actions necessary to achieve the principal’s objectives, based on the agent’s reasonable understanding of them.

Apparent authority focuses on the reasonable belief of the third party based on the principal’s behavior.

299
Q

How are payments made in an installment contract that is silent on time and place of payment?

A

Under the UCC, an installment contract is defined as one in which the goods are to be delivered in multiple shipments, and each shipment is to be separately accepted by the buyer.

Parties cannot vary or contract out of this definition under the code.

Payment by the buyer is due upon each delivery, unless the price cannot be apportioned. When the contract is silent as to when payment is due, the UCC provisions make payment due upon each delivery.

300
Q

What is required to be an accessory after the fact?

A

An accessory after the fact is a person who aids or assists a felon in avoiding apprehension or conviction after commission of the felony.

An accessory after the fact must know that a felony was committed, act specifically to aid or assist the felon, and give the aid or assistance for the purpose of helping the felon avoid apprehension or conviction.

Example of conduct that is not an accessory after the fact:

Here, the woman did not affirmatively act to hinder the man’s capture or arrest or otherwise to give aid to the man, and did not act intending to help him.

Knowledge that a felony has been committed and that a particular person has committed the felony is not sufficient make a person an accessory after the fact.

The mere failure to give information regarding a felony is not sufficient to make a person an accessory after the fact to the felony.

301
Q

Is a defendant’s right to a speedy trial violated when the defendant is given a state appointed defense attorney?

A

NO. Delay in proceeding to trial attributable to the defendant’s attorney is not allocated to the state simply because the attorney was provided by the state to the defendant.

Instead, as with delay attributable to a defendant-supplied attorney, delay attributable to a public defendant is generally treated as caused by the defendant rather than the state.

302
Q

What is embezzlement?

A

Embezzlement is the:

(i) fraudulent
(ii) conversion
(iii) of the property
(iv) of another
(v) by a person who is in lawful possession of the property.

Example:

Here, although the businesswoman owed the independent contract for work that he had done, she was not in possession of the independent contractor’s property. Consequently, she cannot properly be charged with embezzlement

303
Q

What four factors are used to determine the “curtilage” of a home for purposes of Fourth Amendment protection?

A

In addition to the home itself, an area immediately surrounding the home known as the “curtilage” may be covered by the “umbrella” of the Fourth Amendment protection of a home.

In determining whether the area is protected, the following four-factor test applies:

(i) the proximity of the area to the home,
(ii) whether the area is included within an enclosure surrounding the home,
(iii) the nature of the uses to which the area is put, and
(iv) the steps taken by the resident to protect the area from observation by passersby.

304
Q

Is an expert witness subject to cross-examination?

A

Yes. Even a witness qualified as an expert by the court is subject to cross-examination.

A party can challenge the credibility of an expert witness on cross-examination by attacking the adequacy of the expert’s knowledge, both their general knowledge in their field of expertise and their specific knowledge of the facts underlying their testimony.

A party is permitted to question the expert experience and knowledge on cross-examination so the jury can judge the credibility of his testimony, and the attorney may do so by asking about a specific instance of conduct.

305
Q

For purposes of larceny, are unharvested crops considered real-property?

A

Yes. The taking of real-property items (e.g., unharvested crops) is not larceny when the defendant’s act of severance occurs immediately before the carrying away of the real-property items.

The act of harvesting fruit constitutes the severance of real, rather than personal property.

306
Q

False Imprisonment

A

False imprisonment results when a defendant acts:

(i) intending to confine or restrain the plaintiff within boundaries fixed by the defendant,
(ii) those actions directly or indirectly result in such confinement, and
(iii) the plaintiff is conscious (aware) of the confinement or is harmed by it.

Example:

The man accepted her offer for a ride home. Exhausted from his walk, the man immediately fell asleep in the car. The acquaintance decided to continue to the fast food restaurant before taking the man home. After picking up her lunch, she drove the man home and woke him up. The total time the man was in her car did not exceed 15 minutes.

Here, the man was not harmed by the confinement. Thus, to recover for false imprisonment, the man must have been aware of his confinement. However, the man was not aware, as he slept through the trip to the fast food restaurant. Therefore, the man cannot successfully sue for false imprisonment.

307
Q

Wild Animal

A

A wild animal is an animal that is not by custom devoted to the service of humankind in the place where it is being kept.

The possessor of a wild animal is strictly liable for harm done by that animal, in spite of any precautions the possessor has taken to confine the animal or prevent the harm, as long as the plaintiff did not knowingly do anything to bring about the injury and the harm arises from a dangerous propensity that is characteristic of such wild animals or of which the owner has reason to know.

308
Q

Probable cause

Facts supporting probable cause may come from any of the following sources:

A

Facts supporting probable cause may come from any of the following sources:

i) A police officer’s personal observations;
ii) Information from a reliable, known informant or from an unknown informant that can be independently verified; or

309
Q

Life estate pur autre vie

A

A life estate measured by the life of a third party is also called a “life estate pur autre vie.”

Example:

Here, the uncle granted his nephew’s daughter a life estate pur autre vie based on the life of her father, the uncle’s nephew. Upon the nephew’s death, his daughter’s life estate in the oceanfront property terminated.

Because the deed did not specify a subsequent taker of the property, ownership of the oceanfront property reverted to the uncle

310
Q

Profit à prende

A

A profit à prende is a nonpossessory right to enter another’s land and remove specific natural resources.

As a property right, a profit is subject to the applicable recording act.

Profits are created and analyzed similarly to easements, except that profits cannot be created by necessity.

Although a profit can be either exclusive or non-exclusive, most are construed as non-exclusive. If the profit is exclusive, then the holder of the profit has an unlimited and exclusive right to take the subject matter of the profit from the land.

If the profit is non-exclusive, then the right to take the natural resource is either limited by quantity, time, or use, or it is shared with another. Exclusive rights may be assigned or otherwise transferred and apportioned, as long as the apportionment is not wholly inconsistent with the original agreement.

Non-exclusive rights can be assignable or otherwise transferred, but apportionment is not permitted when the burden on the servient estate is increased.

Under the “one stock” rule, the transferees are limited to the amount of material taken by the transferor (i.e., his “stock”), and this quantity is divided up by transferees taking the profit.

311
Q

Can the credibility of a witness be bolstered?

A

The credibility of a witness may not be bolstered. Evidence of the truthful character of the witness is admissible only after the witness’s character for truthfulness has been attacked.

Evidence that impeaches the witness but does not specifically attack the witness’s character for truthfulness, such as testimony that the witness is biased, does not constitute an attack.

312
Q

Can you defend against a felony-murder charge by establishing a valid defense to the underlying felony?

A

Yes. One way to defend against a felony-murder charge successfully is to establish a valid defense to the underlying felony.

Example:

Here, the underlying felony is the robbery, to which the defendant may raise the affirmative defense of duress.

A defendant may raise the affirmative defense of duress if a third party’s unlawful threat causes a defendant to reasonably believe that the only way to avoid death or serious bodily injury to himself or another is to violate the law, and that belief causes the defendant to violate the law.

313
Q

Does the right to counsel exist when the defendant is not sentenced to incarceration but just given a fine?

A

The Sixth Amendment provides a constitutional right to counsel in any case in which the defendant is sentenced to incarceration, even if that sentence is suspended.

The right to counsel, on the other hand, exists only when the defendant is actually sentenced to incarceration, regardless of the maximum length of the sentence that may be imposed for the crime.

314
Q

Private Nuisance

A

A private nuisance is a substantial and unreasonable interference with the use or enjoyment of another individual’s property.

An interference is substantial if it would be offensive, annoying, or inconvenient to a normal, reasonable person in the community.

A person who is not offended, inconvenienced, or annoyed is nevertheless entitled to recover if a normal person would be.

315
Q

Must a a federal law or regulation that expressly preempts state law be narrowly construed or broadly construed?

A

A federal law or regulation that expressly preempts state law must be narrowly construed, not broadly construed.

316
Q

If a court has taken judicial notice of a fact, what must the judge instruct the jury in civil cases vs. criminal cases?

A

Instructing the jury

1) Civil case

In a civil case, the jury must be instructed to accept the noticed fact as conclusive. Fed. R. Evid. 201(f).

2) Criminal case

In a criminal case, the jury must be instructed that it may or may not accept any judicially noticed fact as conclusive. Fed. R. Evid. 201(f).

Judicial notice is the court’s acceptance of an adjudicative fact as true without requiring formal proof.

  • A fact that can be accurately and readily determined need not be generally known as long as it can be determined from a source whose accuracy cannot be reasonably questioned.
317
Q

If a lease agreement contains a provision prohibiting the assignment of the lease without the landlord’s permission, does this provision apply to both a sub-lease and assignment or just the assignment?

A

Just the assignment.

An assignment is a complete transfer of the tenant’s remaining lease term.

A tenant is free to sublet the leasehold unless the landlord and tenant agree otherwise. Any lease clause that purports to limit this right is narrowly construed.

Example:

a) Although the lease prohibited the assignment of the leasehold without the prior written permission of the owner, it did not restrict the physician’s right to sublet the leasehold.
b) Although the owner did accept rental payments from the accountant, the accountant was not an assignee, but only a sublessee because the accountant rented the office space from the realtor for only a one-year period rather than for the full remaining term of the lease between the realtor and the owner. Consequently, the owner’s action in accepting rental payments from the accountant did not constitute a waiver of the assignment-restriction provision.

318
Q

Merchant’s Firm Offer Rule

A

Under the UCC merchant’s firm offer rule, an offer to buy or sell goods is irrevocable, even though consideration is not received to keep the offer open, if:

(i) the offeror is a merchant,
(ii) there is an assurance that the offer is to remain open, and
(iii) the assurance is contained in a signed writing from the offeror.

319
Q

Expectation Damages Example

what is the least likely to recover?

A

“Expectation damages must be foreseeable, and because the landowner and the artist contracted only for the artist to paint murals, the sculpted hanging ceramic wall plaques inspired by each cabin’s theme would be an unforeseeable expense.”

Nonbreaching parties may often choose between several types of damages, including reliance damages (those the plaintiff reasonably incurred in reliance upon the contract) and expectation damages (which put the nonbreaching party in the position that she would have been in if not for the breach).

To calculate expectation damages, compare the value of performance without the breach (what was promised) with the value of the performance with the breach (what was received).

320
Q

The Section 5 Enabling Clause of the Fourteenth Amendment permits Congress to pass legislation to enforce the _______________?

A

The Section 5 Enabling Clause of the Fourteenth Amendment permits Congress to pass legislation to enforce…

the equal protection and due process rights guaranteed by the Fourteenth Amendment, as long as there is “congruence and proportionality” between the injury to be prevented or remedied and the means adopted to achieve that end.

Congress may override state government action that infringes upon Fourteenth Amendment rights if the “congruence and proportionality” test is satisfied, but it may not regulate wholly private conduct under this amendment.

321
Q

What controls:

Offeree first sends acceptance and later sends rejection?

A

If the offeree sends an acceptance and later sends a communication rejecting the offer, then the acceptance will generally control even if the offeror receives the rejection first.

If, however, the offeror receives the rejection first and detrimentally relies on the rejection, then the offeree will be estopped from enforcing the contract.

Example:

In this case, the athlete mailed his acceptance and then mailed a rejection of the president’s offer. Although the president received the rejection first, the athlete’s acceptance would generally control as it was effective when sent.

Under the mailbox rule, because an acceptance becomes valid when sent, a properly addressed letter sent by the offeree operates as an acceptance when mailed, even though the offeror has not yet received the notice.

However, the president detrimentally relied on the athlete’s rejection by booking a different speaker at a more expensive rate. Therefore, the athlete will be estopped from enforcing the contract with the sales company.

322
Q

Can a juror testify about a statement made or any incident that occurred during the jury’s deliberations?

A

NO.

During an inquiry into the validity of a verdict, a juror generally MAY NOT testify about any statement that was made or any incident that occurred during the jury’s deliberations, such as a refusal to apply the court’s instructions.

323
Q

When can a non-freehold tenant remove a fixture that the tenant has attached to the leased property?

A

Absent an agreement to the contrary, a non-freehold tenant can remove a fixture that the tenant has attached to the leased property if:

(i) the leased property can be and is restored to its former condition after the removal, and
(ii) the removal and restoration is made within a reasonable time.

324
Q

Can a party who has not submitted a prior motion for JMOL submit such a motion after the jury has rendered its verdict?

A

NO.

A party who has not submitted a prior motion for judgment as a matter of law may not submit such a motion after the jury has rendered its verdict.

After the jury has rendered its verdict, a party may only renew a motion for judgment as a matter of law that it has previously made.

325
Q

If a party files a claim, a counterclaim, or a cross-claim against another party, must the second party file any claim that that party has against the first party that arises out of the same transaction or occurrence as the first party’s claim?

A

YES. If a party files a claim, a counterclaim, or a cross-claim against another party, the second party must file any claim that that party has against the first party that arises out of the same transaction or occurrence as the first party’s claim.

Example:

The first defendant is an opposing party with respect to this mandatory claim; accordingly, the second defendant’s claim is a compulsory counterclaim.

(Note: A cross-claim may properly be pursued in the current action only if it arises out of the same transaction or occurrence as the original complaint. Consequently, a claim made in response to a cross-claim that arises out of the same transaction or occurrence as the original complaint also arises out of the same transaction or occurrence as the cross-claim.)

326
Q

Does common law conspiracy require an overt act?

A

NO.

At common law, all that is required for a conspiracy to be established is:

1) an agreement between two or more persons
2) to accomplish an unlawful purpose
3) with the intent to accomplish that purpose.

No overt act is required for the conspiracy to be complete at common law.

327
Q

If only one of multiple co-conspirators is charged with conspiracy, does that prevent a conviction of the conspirator who is charged?

A

NO.

The fact that all but one of the co-conspirators are not charged with conspiracy does not prevent a conviction of the conspirator who is charged, so long as the prosecution can show that there was an agreement to accomplish the unlawful act, with the intent to accomplish the act.

328
Q

When does the Pinkerton Rule say?

A

The “Pinkerton Rule” says that every co-conspirator is guilty of any substantive offense committed in furtherance of the conspiracy, regardless of actual knowledge of its commission.

329
Q

Is a threat to damage or destroy property, other than the victim’s home, sufficient to satisfy the “by force or intimidation” element of robbery?

A

Probably not.

Robbery is larceny by force or intimidation when the taking of the property is from the person or presence of the victim.

Example:

Although the phone was still legally within the wrestler’s presence after he dropped it, the taking of property by intimidation must be the result of a threat of immediate serious physical injury to the victim, a close family member, or other person present. The woman just threatened to key his car and break the mirrors.

A threat to damage or destroy property, other than the victim’s home, is probably not sufficient.

330
Q

Wills / Conflict of laws

Which state’s laws govern the distribution of person property vs. real property?

A

Questions regarding the transfer of personal property from someone who dies intestate or who has a will are governed by the law of the deceased’s domicile at the time of death.

Questions regarding the transfer of real property from someone who dies intestate or who has a will are governed by the law of the situs.

Example:

Here, Zach died a domiciliary of State A.

At the time of his death, Zach owned a house in State A, he owned a farm in State B and had a savings account at a bank in State B.

Accordingly, State A law applies in the disposition of Zach’s house and State B law applies in the disposition of Zach’s farm.

Because Zach was domiciled in State A at the time of his death, State A law applies to Zach’s personal property, the bank account even though it is located in State B.

331
Q

Wills

What must be established for an individual to be classified as issue of another?

A

A parent-child relationship must be established for an individual to be classified as issue of another.

Under the UPC and the majority of jurisdictions, adoption establishes a parent-child relationship between the stepparent and child, including full inheritance rights in both directions.

332
Q

Wills

Out-of-wedlock child’s ability to inherit

A

The common-law rule was that if a child was born out of wedlock, he could not inherit from his natural father.

Most jurisdictions provide that an out-of-wedlock child can inherit from his natural father if:

(i) the father subsequently married the natural mother,
(ii) the father held the child out as his own and either received the child into his home or provided support,
(iii) paternity was proven by clear and convincing evidence after the father’s death, OR
(iv) paternity was adjudicated during the lifetime of the father by a preponderance of the evidence.

It has been held unconstitutional to deny inheritance rights to a nonmaterial child when paternity has been established during the father’s lifetime.

333
Q

Per Se Taking Rule Statement

A

Generally, a governmental regulation that adversely affects a person’s property interest is not a taking.

However, it is possible for a regulation to rise to the level of a taking, such as when a regulation results in a permanent physical occupation of the property by the government or a third party or when the regulation results in a permanent or total loss of the property’s economic value.

Example:

Here, the ordinance does not result in a per se taking based on permanent physical occupation because the restaurant, and not a third party, is required to install flood lights.

(NOTE: so if the regulation requires the property owner to install something (i.e., floodlights), but not the government or a third party, then there is not taking.)

Similarly, the ordinance does not result in the total loss of the property’s economic value because it has no impact on the owner’s ability to operate a restaurant. Thus, the ordinance does not amount to a per se taking.

334
Q

Regulatory Taking Rule Statement

A

Regulatory Taking

There is a three-factor balancing test to determine whether an ordinance amounts to a regulatory taking. The following factors are considered:

(i) The economic impact of the regulation on the property owner
(ii) The extent to which the regulation interferes with the owner’s reasonable, investment-backed expectations regarding use of the property, and
(iii) The character of the regulation, including the degree to which it will benefit society, how the regulation distributes the burdens and benefits among property owners, and whether the regulation violates any of the owner’s essential attributes of property ownership, such as the right to exclude others from the property.

335
Q

Can a local government exact promises from a developer, such as setting aside a portion of the land being developed for a park in exchange for issuing the necessary construction permits?

Does this violate the Takings Clause?

A

YES. And such exactions do not violate the Takings Clause if there is:

(i) an essential nexus between legitimate state interests and the conditions imposed on the property owner (i.e., the conditions substantially advance a legitimate state interest), and
(ii) a rough proportionality between the burden imposed by the conditions on property owner and the impact of the proposed development.

In determining whether there is rough proportionality between the burden and the impact, the government must make an individualized determination that the conditions are related both in nature and extent to the impact.

Example:

Here, the city can likely meet the nexus requirement. The legitimate state interest is preventing crime and increasing public safety. The restaurant is seeking to increase the capacity of the restaurant which will increase the economic activity in the area and might attract additional crime. The installation of video surveillance is, therefore, sufficiently related to the city’s legitimate interest.

However, the city will likely fail to meet the rough proportionality requirement because it failed to make an individualized determination that the video surveillance is related both in nature and extent to decreasing crime. The city merely speculates that increased economic activity “might” result in increased crime and that the video surveillance “might” alleviate this potential increase in crime. The city has not supported these assertions with any concrete estimates about the increase in crime or the effectiveness of video surveillance.

Therefore, the city likely cannot carry its burden of showing that the exaction is not a taking, and the permit condition likely violates the Fifth Amendment Takings Clause.

336
Q

Contract for the sale of goods rule statement

A

The UCC Article 2 governs transactions involving the sale of goods.

Under the UCC, a contract is formed if both parties intend to enter into a contract and there is a reasonably certain basis for giving a remedy. Other than the identity of the parties and subject matter of the agreement, the quantity is the only term essential to forming the contract. As long as the parties intend to create a contract, the UCC “fills the gap” if other terms are missing—e.g., time or place of delivery.

337
Q

SOF - Sale over $500 Rule Statement

A

An oral contract for the sale of goods is valid and enforceable unless the contract is for the sale of goods for $500 or more. In that case, the contract must be in writing and signed by the party to be charged (i.e., the one against whom enforcement is sought) in order to satisfy the SOF and be enforceable.

338
Q

When is a UCC contract outside the SOF?

A

A contract for the sale of goods is outside the UCC statute of frauds:

1) to the extent the goods are received and accepted, and
2) to the extent that payment has been made and accepted.

Example:

Here, six knives were delivered to the chef pursuant to the parties’ agreement. The chef delivered payment for those six knives, and the merchant accepted that payment. However, this will only remove the contract from the SOF (i.e., this sale cannot be challenged due to the lack of a writing) with respect to the six knives received by the chef and for which payment was made. Therefore, a writing is still required to enforce the parties’ agreement as to the last four knives.

339
Q

UCC Memorandum for the sale of goods $500 or more

A

The UCC requires a memorandum for a sale of goods for $500 or more to:

(i) indicate that a contract has been made,
(ii) identify the parties,
(iii) contain a quantity term, and
(iv) be signed by the party to be charged.

A “signature” is any authentication that identifies the party to be charged—e.g., a letterhead on the memorandum.

A mistake in the memorandum or the omission of other terms does not destroy the memorandum’s validity. An omitted term can be proved by parol evidence.

However, enforcement of the agreement is limited to the quantity term actually stated in the memorandum.

340
Q

Who is a merchant?

A

A merchant is a person who regularly deals in the type of goods involved in the transaction or otherwise by his occupation holds himself out as having knowledge or skill peculiar to the practices or goods involved in the transaction.

In contracts between merchants for the sale of goods for $500 or more, if a memorandum sufficient against one party is sent to the other party who has reason to know its contents, and the receiving party does not object in writing within 10 days, then the contract is enforceable against the receiving party even though he has not signed it.

Example:

Both the manufacturer and the chef are merchants; the manufacturer regularly deals in knives, and the chef has knowledge and skill particular to chef’s knives based on his occupation. After receiving and paying for six knives, the chef sent a note to the manufacturer on May 17 requesting the remaining four knives.

The manufacturer did not object to this note within ten days of its receipt. Therefore, if the chef’s note were sufficient under the SOF to be enforced against the chef, it could be used to enforce the agreement against the manufacturer for the last four knifes. However, this note was not signed and was not on letterhead, and therefore was not a writing which would have satisfied the SOF if the document needed to be enforced against the chef.

For this reason, the unsigned note would not serve as merchant’s confirmatory memo, and the agreement appears to be unenforceable by the chef as to the last four knives.

341
Q

A firm offer form prepared by the offeree rule statement

A

In general, an offer can be revoked by the offeror at any time prior to acceptance. A promise to hold an offer open is governed by statute.

Under the UCC firm offer rule, an offer to buy or sell goods is irrevocable if:

(i) the offeror is a merchant,
(ii) there is an assurance that the offer is to remain open, and
(iii) the assurance is contained in a signed writing from the offeror.

A firm offer in a form prepared by the offeree, however, must be separately authenticated by the offeror to protect against inadvertent signing.

Finally, it is still possible for an offer to be irrevocable if the offeree reasonably and detrimentally relies on the offeror’s promise prior to acceptance.

  • It must have been reasonably foreseeable that such detrimental reliance would occur in order to imply the existence of an option contract.

Example:

Here, although the offer was in writing and signed by the gardener, because the cook prepared the offer for the gardener’s signature, the gardener must also separately authenticate the 14-day option.

The facts are not clear as to whether the assurance itself is separately signed or initialed. Further, the facts indicate that the gardener had never sold tomatoes before, which indicates the gardener is not a merchant for the purposes of the UCC. Therefore, the offer does not meet the UCC firm offer requirement to make the offer irrevocable.

Here, there are no facts indicating that the gardener should have known the offer would induce action or forbearance in the cook and the gardener was indeed entitled to revoke the offer at any time.

342
Q

Revocation under the UCC Rule Statement

When does revocation become effective?

A

An offer is revoked when the offeror makes a manifestation of an intention not to enter into the proposed contract before the offeree accepts.

A revocation may be made in any reasonable manner and by any reasonable means, and it is not effective until communicated.

Under the UCC, a person receives notice of revocation when:

(i) it comes to that person’s attention, or
(ii) it is duly delivered in a reasonable form at the offeree’s place of business.

Example:

Here, the gardener manifested an intention not to enter into the contract with the cook when he informed her that he could not sell to her because he had sold to someone else.

He communicated the revocation by reasonable means (it is not an issue that the revocation was not in writing) and did so before the cook had accepted the offer. Therefore, the gardener revoked the offer and prevented formation.

343
Q

In a contract for the sale of goods, if no UCC rule applies, what law governs?

A

A contract that involves the sale of a good (e.g., a lawn mower), the contract is governed by the UCC if a UCC rule is applicable.

Where no UCC rule applies, the common law governs the contract.

344
Q

Offer & Acceptance Rule Statement

A

An offer is an objective manifestation of a willingness by the offeror to enter into an agreement that creates the power of acceptance in the offeree. An offer can only be accepted while it remains open.

One way that an offer terminates is by the offeree’s rejection of the offer. After an initial rejection, the offeror can renew his offer.

Example:

  • In this case, the homeowner made a valid offer when he offered to sell his lawn mower to his neighbor for $350. The neighbor rejected, and therefore terminated, this initial offer when she said “No way!” However, the homeowner renewed his offer when he told the neighbor that he was still willing to sell the lawn mower to her for $350. It is this second offer that is at issue.
345
Q

Common law enforceable option to keep open

A

In general, an offer can be revoked by the offeror at any time prior to acceptance. However, an enforceable option will render the offer irrevocable.

An option is an independent promise to keep an offer open for a specified period of time.

Such a promise limits the offeror’s power to revoke the offer until after the period has expired, while also preserving the offeree’s power to accept.

Under the common law, if the option is a promise not to revoke an offer to enter a new contract, the offeree must generally give separate consideration for the option to be enforceable.

346
Q

How does the Merchant Firm Offer Rule exand the definition of “who is a merchant”?

A

A merchant is generally described as a person who regularly deals in the type of goods involved in the transaction.

Under the firm offer rule, a merchant includes not only a person who regularly deals in the type of goods involved in the transaction, but also any businessperson when the transaction is of a commercial nature.

347
Q

Constructive Revocation

A

An offer can only be accepted while it is still outstanding. An offer can be terminated in multiple ways, including revocation.

In general, an offeror may revoke his offer at any time prior to acceptance. An offer is revoked when the offeror makes a manifestation of an intention not to enter into the proposed contract.

A revocation may be made in any reasonable manner and by any reasonable means, and it is not effective until communicated.

Alternatively, if the offeree acquires reliable information that the offeror has taken definite action inconsistent with the offer, the offer is automatically revoked. This is called constructive revocation.

Example:

Here, the homeowner did not manifest his intention to revoke the offer directly to the neighbor. However, there was a constructive revocation. The neighbor received a call from an acquaintance saying that he had purchased the lawn mower from the homeowner. The acquaintance also showed the neighbor the signed contract between the acquaintance and the homeowner. This amounts to reliable information and selling the lawn mower to someone else is certainly inconsistent with the homeowner’s offer to the neighbor.

348
Q

The business judgment rule and a conflict-of-interest transaction & Business judgment rule safe harbor provision

A

The business judgment rule is a rebuttable presumption that a director reasonably believed that his actions were in the best interests of the corporation. The exercise of managerial powers by a director is generally subject to the business judgment rule. However, the business judgment rule does not generally apply to a conflict-of-interest transaction.

A conflict-of-interest transaction, or “self-dealing,” is any transaction between a director and his corporation that would normally require approval of the board of directors and that is of such financial significance to the director that it would reasonably be expected to influence the director’s vote on the transaction.

The interest involved can be direct or indirect, but it must be financial and material. _Majority approval of a conflict-of-interest transaction by fully informed *disinterested* *directors* *triggers* the business judgment rule under a safe harbor provision_.

Example:

Here, the corporation’s sale of the tower involved self-dealing. As owners of the LLC, the four directors of the corporation had a one-fourth financial interest in the LLC. These interests would reasonably be expected to influence their votes in their capacities as corporation directors. The transaction was not authorized by a majority of informed and disinterested directors because the only directors who voted on the transaction also held a financial stake in the purchasing LLC. Therefore, the safe-harbor provision does not apply, and the business judgment rule does not apply.

349
Q

Directors of a Corporation duties of loyalty and care

Conflict of interest and safe harbor requirements

Fairness Test

A

A director owes a duty of loyalty and a duty of care to the corporation. The duty of loyalty requires a director to act in a manner that the director reasonably believes is in the best interests of the corporation.

A director who engages in a conflict-of-interest transaction with his own corporation has violated his duty of loyalty unless the transaction is protected under the safe-harbor rule.

A conflict-of-interest transaction may enjoy safe-harbor protection via:

(i) a majority vote of informed and disinterested directors;
(ii) a majority vote of informed and disinterested shareholders; or
(iii) fairness of the transaction.

The fairness test looks at the substance and procedure of the transaction.

Substantively, the test asks whether the corporation received something of comparable value in exchange for what it gave to the director.

Procedurally, it looks at whether the process followed by the directors in reaching their decision was appropriate.

The interested directors have the burden of establishing both the substantive and procedural fairness of the transaction.

A conflict-of-interest transaction in violation of the safe-harbor provisions may be enjoined or rescinded, and the corporation may seek damages from the directors.

350
Q

Is a statement describing past or present symptoms hearsay if it is made for medical diagnosis or treatment?

A

A statement describing past or present symptoms is not hearsay if it is made for medical diagnosis or treatment.

A statement of the cause or source of the condition is admissible if it is reasonably pertinent to diagnosis or treatment. The statement need not be made to a physician nor be made by the patient.

351
Q

Confrontation Clause Rule Statement

A

Under the Sixth Amendment Confrontation Clause, a criminal defendant has the right to confront any witnesses against him.

An out-of-court testimonial statement is not admissible against a criminal defendant when the declarant is unavailable to testify unless the defendant had a prior opportunity to cross-examine the declarant.

However, a statement that is not testimonial does not violate the Confrontation Clause.

In determining whether a statement is testimonial, an objective analysis of the circumstances is key, and courts look to the primary purpose of the statement.

  • A statement made to police with the primary purpose of enabling police to help meet an ongoing emergency (e.g., during a 911 call) is not testimonial.
352
Q

What are the requirements necessary to admit a testimonial hearsay statement against a criminal defendant?

A

In order to admit a testimonial hearsay statement against a criminal defendant,

(i) the declarant must be unavailable to testify, and
(ii) the defendant must have had a prior opportunity to cross-examine the declarant.

A statement made during a police interrogation that has the primary purpose of ascertaining criminal conduct is testimonial under the Confrontation Clause.

353
Q

Admissibility of Subsequent Remedial Measures

A

When a party takes subsequent measures that would have made the other party’s earlier injury or harm less likely to have occurred, evidence of these subsequent measures is not admissible to prove negligence.

However, evidence of subsequent remedial measures may be admissible for another purpose—e.g.,

1) impeachment,
2) ownership, or
3) control of the cause of the harm, or
4) the feasibility of precautionary measures.

Example:

Here, though relevant, evidence of the hospital’s change in policy (i.e., a remedial measure) made subsequent to the woman’s treatment is not admissible to prove negligence. Nothing in the facts suggests that the evidence is being offered for any reason other than to prove negligence. Therefore, the court should grant the hospital’s motion to exclude evidence of the hospital’s new policy.

354
Q

Compromise offer admissibility rule statement

A

Compromise offers made by any party are not admissible to prove the validity of a disputed claim, nor are they admissible for impeachment by prior inconsistent statement or contradiction.

Example:

Here, evidence of the hospital’s offer to settle the woman’s claim is relevant because it suggests that it is more probable that it was negligent in its treatment of the woman. But the hospital denied culpability in response to the woman’s complaint placing the validity of her claim in dispute.

Therefore, the hospital’s subsequent offer to compromise this disputed claim is not admissible and the court should grant the hospital’s motion to exclude this evidence.

355
Q

Admissibility of offers to pay medical expenses

A

Evidence of compromise offers made by a party are not admissible to prove the validity of a disputed claim. Similarly, evidence of the payment, offer to pay, or promise to pay medical or similar expenses resulting from an injury is not admissible to prove liability for the injury.

However, unlike a compromise offer, the validity or amount of a claim need not be in dispute for an offer to pay medical expenses to be inadmissible.

Example:

Here, the man texted his offer of $10,000 one week after the alleged incident. The woman filed her claim nearly four months later. As a result, the man offered the woman $10,000 before there was any disputed claim between them. His statement is both relevant and not excluded under the compromise offer rule.

By contrast, the man’s offer to pay the woman’s medical expenses, though relevant, is inadmissible regardless of when their dispute arose.

Therefore, the court should deny the man’s motion to exclude the offer of compromise and grant the man’s motion to exclude the evidence of his offer to pay the woman’s medical expenses.

356
Q

“Rape Shield” Law Rule Statement

A

Pursuant to the “rape shield” rule, evidence offered to prove the sexual behavior or predisposition of an alleged victim of sexual assault is generally inadmissible in both civil and criminal proceedings.

However, in a civil case, evidence offered to prove an alleged victim’s sexual behavior or predisposition is admissible if its probative value substantially outweighs the danger of harm to any victim and of unfair prejudice to any party.

Example:

Here, the woman has alleged that her injuries were especially traumatic because of her belief in sexual abstinence before marriage and her lack of prior sexual experience.

Consequently, the woman’s prior sexual conduct has great probative value in determining the extent of her injury.

On the other hand, this evidence is not likely to harm the woman or unfairly prejudice her sexual assault claim. Therefore, the court would likely grant the man’s motion to admit the evidence.

357
Q

MPT Testable Areas

A
  1. Factual Analysis

When testing on this area, the examiners will give you one of three possible tasks:

(i) Draft an opening statement for trial;
(ii) Draft a closing argument for trial; or
(iii) Draft jury instructions.
2. Fact Gathering

You will recognize this area because the examiners will present you with your client’s case, and ask you what additional factual evidence you need to gather to improve your client’s position.

  1. Legal Analysis and Reasoning

You should demonstrate the ability to analyze statutory, case and administrative materials for relevant principles of law and apply them to specific factual situations. Think back to first-year legal research and writing. Legal analysis and reasoning includes the ability to:

i) identify and formulate legal issues;
ii) identify relevant legal rules within a given set of legal materials;
iii) formulate relevant legal theories;
iv) elaborate legal theories; evaluate legal theories; and
v) criticize and synthesize legal argumentation.
4. Problem Solving

You will recognize this area because it is the only MPT problem that asks you to go beyond the legal solutions to your client’s problem by considering non-legal solutions.

You should demonstrate the ability to develop and evaluate strategies for solving a problem or accomplishing the objectives of your client. The problem-solving process includes the ability to identify and diagnose the problem, generate alternative solutions and strategies, and recommend a solution.

  1. Recognition and Resolution of Ethical Dilemmas

You will recognize this area because when testing on ethical dilemmas, the examiners must include in the problem the code of ethics for lawyers in the relevant jurisdiction.

358
Q

Common MPT documents to draft

A
  1. Memorandum of Law
  2. Pleadings
    a) Complaint
    b) Answer
  3. Motions
    a) Motion to Dismiss
    b) Motion for Summary Judgment
    c) Motion in Limine

This is persuasive pretrial motion that requests the court to issue an interlocutory order that prevents an opposing party from introducing or referring to potentially irrelevant, prejudicial, redundant, or otherwise inadmissible evidence until the court has ruled on its admissibility.

  1. Writings to Parties
    a) Letters to Client
    b) Letters to Opposing Counsel
  2. Other documents
    a) Persuasive briefs
    b) Position Papers
    c) Statement of Facts
    d) Discovery Requests
    e) Opening Statements and closing arguments
359
Q

General Partnership Rule statement

A

A general partnership is an association of two or more persons to carry on a for-profit business as co-owners. It is not necessary that such persons have the specific intent to form a partnership.

The only agreement necessary to create a partnership is the agreement to conduct a for-profit business as co-owners. This agreement may even be implied by the conduct of the parties when they have not entered into a written or oral agreement.

360
Q

When does a partner contractually bind the partnership?

A

A partner is an agent of the partnership for the purpose of its business and can contractually bind the partnership when the partner acts with either actual or apparent authority.

Acting individually, a partner has the actual authority to commit the partnership to usual and customary matters, unless the partner has reason to know that other partners might disagree.

On the other hand, each partner has equal rights in the management and conduct of the partnership’s business.

If there is a decision as to a matter outside the ordinary course of the partnership’s business, the decision requires the consent of all partners.

Example:

As a partner, Carol had the authority to purchase the imaging machine on behalf of the partnership without the consent of either Jean or Pat because the purchase was within the course of ordinary business. The partners had purchased state-of-the-art imaging equipment when they started the business, they agreed to run the practice in a manner consistent with other area practices, and the equipment was the same equipment that other practices in the area had purchased.

Thus, purchasing the new machine is consistent with the partnership’s ordinary business.

361
Q

What is the effect of a partner’s express will to withdraw?

A

A partner’s notice to the partnership of the partner’s express will to withdraw can trigger a partner’s dissociation from the partnership.

  • A partnership at will is dissolved when a partner chooses to dissociate from the partnership by giving notice of her withdrawal.
  • A partner has the power to dissociate from the partnership at any time, even if the dissociation is wrongful.*

A partnership agreement cannot prevent a partner from withdrawing from the partnership, but it can require that the partner’s notice of withdrawal be in writing.

For a partnership that is unlimited by time or undertaking, a partner’s dissociation is wrongful only when it is in breach of an express provision of the partnership agreement.

Example:

Here, Jean’s oral statement to Carol that she was “out of here and never coming back” constituted a valid dissociation because she gave notice of her withdrawal to Carol and Pat.

The dissociation was not wrongful because this is an at-will partnership and there is no indication that withdrawal breached any express provision in the partnership agreement.

362
Q

When is a partner’s dissociation wrongful?

A

For a partnership that is unlimited by time or undertaking, a partner’s dissociation is wrongful only when it is in breach of an express provision of the partnership agreement.

363
Q

What must all partners (including any properly dissociated partners) agree to in order to carry on its business after a partnership has been dissovled but before the winding up is complete?

A

Once a partnership has been dissolved, but before the winding up of its business is complete, the partnership may resume carrying on its business as if dissolution had never occurred.

To do so, all partners (including any properly dissociated partners) must agree to waive the right to terminate the partnership.

When a partner dissociates from the partnership but the partnership is not dissolved, the partnership must buy out the dissociated partner’s partnership interest.

A dissociated partner may maintain an action against the partnership to determine the buyout price and to compel the partnership to pay that amount to the partner.

The action must be commenced within 120 days after the partnership has tendered payment or an offer to pay or within one year after written demand for payment if no payment or offer to pay is tendered.

Example

In this case, Carol and Pat agreed to continue their participation in Radiology Services without Jean, however it does not appear that Jean, the dissociated partner, agreed to waive the right to terminate the partnership with them. Therefore, the partnership remained dissolved, and Jean cannot receive a buyout payment.

On the other hand, if Jean agreed to waive the right to terminate the partnership, then Radiology Services must buy out Jean’s partnership interest. Jean asked Carol to give Jean her share of the value of the practice, and she can maintain an action against Radiology Services to determine the buyout price and compel it to pay that amount to her. She must commence this action within 120 days after Radiology Services has tendered payment or an offer to pay.

364
Q

What is the modern trend for a lease provision that prevents assignment without the LL’s permiission but is silent as to a standard for exercising that persmission?

A

Absent any language to the contrary, a lease can be freely assigned. When a lease prohibits the tenant from assigning the lease, the tenant may nevertheless assign; however, the landlord generally can then terminate the lease for breach of one of its covenants and recover any damages.

When a lease prevents assignment without the permission of the landlord, and the lease is silent as to a standard for exercising that permission, the modern trend imposes a requirement that the landlord may withhold permission only on a reasonable ground in relation to the property being leased and not on a whim or personal prejudice.

The traditional rule is that the landlord may withhold permission at his discretion.

Examples:

The tenant will argue that, although the landlord’s consent to an assignment was required by the lease, his refusal to rent to a tenant in good financial standing is unreasonable and evidence of a lack of good faith.

The landlord will counter, however, that the lease provision that restricted assignments without his consent was valid and that he could refuse to accept the tenant’s proposed assignee under that clause for any reason. Further, even if this is a jurisdiction that requires the landlord’s refusal to be reasonable, he will also argue that, given his previous experience with lawyers as tenants, his refusal was also reasonable under the circumstances.

Finally, he immediately rejected the proposed assignment and made clear a new lessee was required.

365
Q

When does a tenant surrender the leasehold?

A

Termination of a tenancy for years may occur before the expiration of the term, such as when the tenant surrenders the leasehold.

A tenant surrenders a lease by transferring the lease back to the landlord with the landlord accepting the return.

If the landlord accepts surrender, the tenant is not obligated for future rent.

366
Q

Does a LL have a duty to mitigate damages following a tenant’s abandonment of the premises?

A

Many states impose on the landlord a duty to make reasonable efforts to mitigate damages when a tenant abandons a lease.

What constitutes a reasonable effort depends on the circumstances, however an owner of multiple vacant apartments is typically required only to treat the premises as one of his vacant stock.

367
Q

Secured Transactions

Attachment Rule Statement

A

Under Article 9, for a security interest to be enforceable against a debtor, the interest must attach to the collateral.

For attachment, three conditions must be met:

(i) value given by the secured party,
(ii) the debtor has rights in the collateral, and
(iii) the debtor authenticated a security agreement that describes the collateral (or the secured party has possession or control of the collateral pursuant to a security agreement).

368
Q

Secured Transactions

After-acquired property rule statement

A

A debtor may also give a security interest in future rights. This type of interest is created by including an “after-acquired property clause” in the security agreement.

The security interest for after-acquired property attaches as soon as the debtor obtains an interest in the property.

369
Q

Secured Transactions

Priority / PMSI

A

In order to have priority over a third party who has a security interest in the same collateral, perfection of the security is generally necessary.

A purchase money security interest (PMSI) is a security interest in goods that has priority over other security interests in the same goods. It arises when a creditor sells goods to a debtor on credit and retains a security interest in those goods, or the creditor advances funds, which are then used to purchase the goods and the creditor reserves a security interest in those goods.

While, generally, a PMSI has priority over other security interests in the same goods, a PMSI in inventory has priority only if:

(i) the PMSI is perfected by the time the debtor receives possession of the collateral; and
(ii) the purchase-money secured party sends an authenticated notification of the PMSI to the holder of any conflicting security interest before the debtor receives possession of the collateral.

A PMSI in goods other than inventory or livestock prevails over all other security interests in the collateral, even if they were previously perfected, if the secured party perfects before or within 20 days after the debtor receives possession of the collateral.

Only purchase-money security interests in consumer goods are perfected upon attachment without the filing of a financing statement.

370
Q

Secured Transactions

A transaction in the form of a lease is treated as a security interest if…

A

A transaction in the form of a lease is treated as a security interest if:

i) the lessee must pay consideration to the lessor for the right to possess and use the goods for the term of the lease,
ii) the payment obligation cannot be terminated by the lessee, and
iii) the lessee has an option to become the owner of the goods upon completion of the lease agreement.